Elementary Number Theory And Primality Tests [unkn] WW

background image

Chapter 1

The Fundamental Theorem of
Arithmetic

1.1

Prime numbers

If a, b

∈ Z we say that a divides b (or is a divisor of b) and we write a | b, if

b = ac

for some c

∈ Z.

Thus

−2 | 0 but 0 - 2.

Definition 1.1 The number p

∈ N is said to be prime if p has just 2 divisors in N,

namely 1 and itself.

Note that our definition excludes 0 (which has an infinity of divisors in N) and

1 (which has just one).

Writing out the prime numbers in increasing order, we obtain the sequence of

primes

2, 3, 5, 7, 11, 13, 17, 19, . . .

which has fascinated mathematicians since the ancient Greeks, and which is the
main object of our study.

Definition 1.2 We denote the nth prime by p

n

.

Thus p

5

= 11, p

100

= 541.

It is convenient to introduce a kind of inverse function to p

n

.

Definition 1.3 If x

∈ R we denote by π(x) the number of primes ≤ x:

π(x) =

k{p ≤ x : p prime}k.

Thus

π(1.3) = 0, π(3.7) = 2.

Evidently π(x) is monotone increasing, but discontinuous with jumps at each

prime x = p.

1–1

background image

374

1–2

Theorem 1.1 (Euclid’s First Theorem) The number of primes is infinite.

Proof

I

Suppose there were only a finite number of primes, say

p

1

, p

2

, . . . , p

n

.

Let

N = p

1

p

2

· · · p

n

+ 1.

Evidently none of the primes p

1

, . . . , p

n

divides N .

Lemma 1.1 Every natural number n > 1 has at least one prime divisor.

Proof of Lemma

B

The smallest divisor d > 1 of n must be prime. For otherwise

d would have a divisor e with 1 < e < d; and e would be a divisor of n smaller
than d.

C

By the lemma, N has a prime factor p, which differs from p

1

, . . . , p

n

.

J

Our argument not only shows that there are an infinity of primes; it shows that

p

n

< 2

2

n

;

a very feeble bound, but our own. To see this, we argue by induction. Our proof
shows that

p

n+1

≤ p

1

p

2

· · · p

n

+ 1.

But now, by our inductive hypothesis,

p

1

< 2

2

1

, p

2

< 2

2

2

, . . . , p

n

< 2

2

n

.

It follows that

p

n+1

≤ 2

2

1

+2

2

+

···+2

n

But

2

1

+ 2

2

+

· · · + 2

n

= 2

n+1

− 1 < 2

n+1

.

Hence

p

n+1

< 2

2

n+1

.

It follows by induction that

p

n

< 2

2

n

,

for all n

≥ 1, the result being trivial for n = 1.

This is not a very strong result, as we said. It shows, for example, that the 5th

prime, in fact 11, is

< 2

2

5

= 2

32

= 4294967296.

In general, any bound for p

n

gives a bound for π(x) in the opposite direction,

and vice versa; for

p

n

≤ x ⇐⇒ π(x) ≥ n.

background image

374

1–3

In the present case, for example, we deduce that

π(2

2

y

)

≥ [y] > y − 1

and so, setting x = 2

2

y

,

π(x)

≥ log

2

log

2

x

− 1 > log log x − 1.

for x > 1. (We follow the usual convention that if no base is given then log x
denotes the logarithm of x to base e.)

The Prime Number Theorem (which we shall make no attempt to prove) asserts

that

p

n

∼ n log n,

or, equivalently,

π(x)

x

log x

.

This states, roughly speaking, that the probability of n being prime is about

1/ log n. Note that this includes even numbers; the probability of an odd number
n being prime is about 2/ log n. Thus roughly 1 in 6 odd numbers around 10

6

are

prime; while roughly 1 in 12 around 10

12

are prime.

(The Prime Number Theorem is the central result of analytic number theory

since its proof involves complex function theory. Our concerns, by contrast, lie
within algebraic number theory.)

There are several alternative proofs of Euclid’s Theorem. We shall give one

below. But first we must establish the Fundamental Theorem of Arithmetic (the
Unique Factorisation Theorem) which gives prime numbers their central rˆole in
number theory; and for that we need Euclid’s Algorithm.

1.2

Euclid’s Algorithm

Proposition 1.1 Suppose m, n

∈ N, m 6= 0. Then there exist unique q.r ∈ N

such that

n = qm + r,

0

≤ r < m.

Proof

I

For uniqueness, suppose

n = qm + r = q

0

m + r

0

,

where r < r

0

, say. Then

(q

0

− q)m = r

0

− r.

The number of the right is < m, while the number on the left has absolute value
≥ m, unless q

0

= q, and so also r

0

= r.

We prove existence by induction on n. The result is trivial if n < m, with

q = 0, r = n. Suppose n

≥ m. By our inductive hypothesis, since n − m < n,

n

− m = q

0

m + r,

background image

374

1–4

where 0

≤ r < m. But then

n = qm + r,

with q = q

0

+ 1.

J

Remark: One might ask why we feel the need to justify division with remainder
(as above), while accepting, for example, proof by induction. This is not an easy
question to answer.

Kronecker said, “God gave the integers. The rest is Man’s.” Virtually all

number theorists agree with Kronecker in practice, even if they do not accept his
theology. In other words, they believe that the integers exist, and have certain
obvious properties.

Certainly, if pressed, one might go back to Peano’s Axioms, which are a stan-

dard formalisation of the natural numbers. (These axioms include, incidentally,
proof by induction.) Certainly any properties of the integers that we assume could
easily be derived from Peano’s Axioms.

However, as I heard an eminent mathematician (Louis Mordell) once say, “If

you deduced from Peano’s Axioms that 1+1 = 3, which would you consider most
likely, that Peano’s Axioms were wrong, or that you were mistaken in believing
that 1 + 1 = 2?”

Proposition 1.2 Suppose m, n

∈ N. Then there exists a unique number d ∈ N

such that

d

| m, d | n,

and furthermore, if e

∈ N then

e

| m, e | n =⇒ e | d.

Definition 1.4 We call this number d the greatest common divisor of m and n,
and we write

d = gcd(m, n).

Proof

I

Euclid’s Algorithm is a simple technique for determining the greatest

common divisor gcd(m, n) of two natural numbers m, n

∈ N. It proves inci-

dentally — as the Proposition asserts — that any two numbers do indeed have a
greatest common divisor (or highest common factor).

First we divide the larger, say n, by the smaller. Let the quotient be q

1

and let

the remainder (all we are really interested in) be r

1

:

n = mq

1

+ r

1

.

Now divide m by r

1

(which must be less than m):

m = r

1

q

2

+ r

2

.

background image

374

1–5

We continue in this way until the remainder becomes 0:

n = mq

1

+ r

1

,

m = r

1

q

2

+ r

2

,

r

1

= r

2

q

3

+ r

3

,

. . .

r

t

−1

= r

t

−2

q

t

−1

+ r

t

,

r

t

= r

t

−1

q

t

.

The remainder must vanish after at most m steps, for each remainder is strictly
smaller than the previous one:

m > r

1

> r

2

>

· · ·

Now we claim that the last non-zero remainder, d = r

t

say, has the required

property:

d = gcd(m, n) = r

t

.

In the first place, working up from the bottom,

d = r

t

| r

t

−1

,

d

| r

t

and d

| r

t

−1

=

⇒ d | r

t

−2

,

d

| r

t

−1

and d

| r

t

−2

=

⇒ d | r

t

−3

,

. . .

d

| r

3

and d

| r

2

=

⇒ d | r

1

,

d

| r

2

and d

| r

1

=

⇒ d | m,

d

| r

1

and d

| m =⇒ d | n.

Thus

d

| m, n;

so d is certainly a divisor of m and n.

On the other hand, suppose e is a divisor of m and n:

e

| m, n.

Then, working downwards, we find successively that

e

| m and e | n =⇒ e | r

1

,

e

| r

1

and e

| m =⇒ e | r

2

,

e

| r

2

and e

| r

1

=

⇒ e | r

3

,

. . .

e

| r

t

−2

and e

| r

t

−1

=

⇒ e | r

t

.

Thus

e

| r

t

= d.

background image

374

1–6

We conclude that our last non-zero remainder r

t

is number we are looking for:

gcd(m, n) = r

t

.

J

It is easy to overlook the power and subtlety of the Euclidean Algorithm. The

algorithm also gives us the following result.

Theorem 1.2 Suppose m, n

∈ N. Let

gcd(m, n) = d.

Then there exist integers x, y

∈ Z such that

mx + ny = d.

Proof

I

The Proposition asserts that d can be expressed as a linear combination

(with integer coefficients) of m and n. We shall prove the result by working
backwards from the end of the algorithm, showing successively that d is a linear
combination of r

s

and r

s+1

, and so, since r

s+1

is a linear combination of r

s

−1

and

r

s

, d is also a linear combination of r

s

−1

and r

s

.

To start with,

d = r

t

.

From the previous line in the Algorithm,

r

t

−2

= q

t

r

t

−1

+ r

t

.

Thus

d = r

t

= r

t

−2

− q

t

r

t

−1

.

But now, from the previous line,

r

t

−3

= q

t

−1

r

t

−2

+ r

t

−1

.

Thus

r

t

−1

= rt

− 3 − q

t

−1

r

t

−2

.

Hence

d = r

t

−2

− q

t

rt

− 1

= r

t

−2

− q

t

(r

t

−3

− q

t

−1

r

t

−2

)

=

−q

t

r

t

−3

+ (1 + q

t

q

t

−1

)r

t

−2

.

Continuing in this way, suppose we have shown that

d = a

s

r

s

+ b

s

r

s+1

.

Since

r

s

−1

= q

s+1

r

s

+ r

s+1

,

background image

374

1–7

it follows that

d = a

s

r

s

+ b

s

(r

s

−1

− q

s+1

r

s

)

= b

s

r

s

−1

+ (a

s

− b

s

q

s+1

)r

s

.

Thus

d = a

s

−1

r

s

−1

+ b

s

−1

r

s

,

with

a

s

−1

= b

s

, b

s

−1

= a

s

− b

s

q

s+1

.

Finally, at the top of the algorithm,

d = a

0

r

0

+ b

0

r

1

= a

0

r

0

+ b

0

(m

− q

1

r

0

)

= b

0

m + (a

0

− b

0

q

1

)r

0

= b

0

m + (a

0

− b

0

q

1

)(n

− q

0

m)

= (b

0

− a

0

q

0

+ b

0

q

0

q

1

)m + (a

0

− b

0

q

0

)n,

which is of the required form.

J

Example: Suppose m = 39, n = 99. Following Euclid’s Algorithm,

99 = 2

· 39 + 21,

39 = 1

· 21 + 18,

21 = 1

· 18 + 3,

18 = 6

· 3.

Thus

gcd(39, 99) = 3.

Also

3 = 21

− 18

= 21

− (39 − 21)

=

−39 + 2 · 21

=

−39 + 2(99 − 2 · 39)

= 2

· 99 − 5 · 39.

Thus the Diophantine equation

99x + 39y = 3

has the solution

x = 2, y =

−5.

(By a Diophantine equation we simply mean a polynomial equation to which we
are seeking integer solutions.)

background image

374

1–8

This solution is not unique; we could, for example, add 39 to x and subtract

99 from y. We can find the general solution by subtracting the particular solution
we have just found to give a homogeneous linear equation. Thus if x

0

, y

0

∈ Z also

satisfies the equation then X = x

0

− x, Y = y

0

− y satisfies the homogeneous

equation

99X + 39Y = 0,

ie

33X + 13Y = 0,

the general solution to which is

X = 13t, Y =

−33t

for t

∈ Z. The general solution to this diophantine equation is therefore

x = 2 + 13t, y =

−5 − 33t

(t

∈ Z).

It is clear that the Euclidean Algorithm gives a complete solution to the general

linear diophantine equation

ax + by = c.

This equation has no solution unless

gcd(a, b)

| c,

in which case it has an infinity of solutions. For if (x, y) is a solution to the
equation

ax + by = d,

and c = dc

0

then (c

0

x, c

0

y) satisfies

ax + by = c,

and we can find the general solution as before.

Corollary 1.1 Suppose m, n

∈ Z. Then the equation

mx + ny = 1

has a solution x, y

∈ Z if and only if gcd(m, n) = 1.

It is worth noting that we can improve the efficiency of Euclid’s Algorithm by

allowing negative remainders. For then we can divide with remainder

≤ m/2 in

absolute value, ie

n = qm + r,

background image

374

1–9

with

−m/2 ≤ r < m/2. The Algorithm proceeds as before; but now we have

m

≥ |r

0

/2

| ≥ |r

1

/2

2

| ≥ . . . ,

so the Algorithm concludes after at most log

2

m steps.

This shows that the algorithm is in class P, ie it can be completed in polyno-

mial (in fact linear) time in terms of the lengths of the input numbers m, n — the
length of n, ie the number of bits required to express n in binary form, being

[log

2

n] + 1.

Algorithms in class P (or polynomial time algorithms) are considered easy or

tractable, while problems which cannot be solved in polynomial time are consid-
ered hard or intractable. RSA encryption — the standard techniqhe for encrypting
confidential information — rests on the belief — and it should be emphasized that
this is a belief and not a proof — that factorisation of a large number is intractable.

Example: Taking m = 39, n = 99, as before, the Algorithm now goes

99 = 3

· 39 − 18,

39 = 2

· 18 + 3,

18 = 6

· 3,

giving (of course)

gcd(39, 99) = 3,

as before.

1.3

Ideals

We used the Euclidean Algorithm above to show that if gcd(a, b) = 1 then there
we can find u, v

∈ Z such that

au + bv = 1.

There is a much quicker way of proving that such u, v exist, without explicitly

computing them.

Recall that an ideal in a commutative ring A is a non-empty subset a

⊂ A

such that

1. a, b

∈ a =⇒ a + b ∈ a;

2. a

∈ a, c ∈ A =⇒ ac ∈ a.

As an example, the multiples of an element a

∈ A form an ideal

hai = {ac : c ∈ A}.

Such an ideal is said to be principal.

background image

374

1–10

Proposition 1.3 Every ideal a

⊂ Z is principal.

Proof

I

If a = 0 (by convention we denote the ideal

{0} by 0) the result is trivial:

a

=

h0i. We may suppose therefor that a 6= 0.

Then a must contain integers n > 0 (since

−n ∈ a =⇒ n ∈ a). Let d be the

least such integer. Then

a

=

hdi.

For suppose a

∈ a. Dividing a by d,

a = qd + r,

where

0

≤ r < d.

But

r = a + (

−q)d ∈ a.

Hence r = 0; for otherwise r would contradict the minimality of d. Thus

a = qd,

ie every element a

∈ a is a multiple of d.

J

Now suppose a, b

∈ Z. Consider the set of integers

I =

{au + bv : u, v ∈ Z}.

It is readily verified that I is an ideal.

According to the Proposition above, this ideal is principal, say

I =

hdi.

But now

a

∈ I =⇒ d | a,

b

∈ I =⇒ d | b.

On the other hand,

e

| a, e | b =⇒ e | au + bv

=

⇒ e | d.

It follows that

d = gcd(a, b);

and we have shown that the diophantine equation

au + bv = d

always has a solution.

In particular, if gcd(a, b) = 1 we can u, v

∈ Z such that

au + bv = 1.

background image

374

1–11

This proof is much shorter than the one using the Euclidean Algorithm; but it

suffers from the disadvantage that it provides no way of computing

d = gcd(a, b),

and no way of solving the equation

au + bv = d.

In effect, we have taken d as the least of an infinite set of positive integers, using
the fact that the natural numbers N are well-ordered, ie every subset S

⊂ N has a

least element.

1.4

The Fundamental Theorem of Arithmetic

Proposition 1.4 (Euclid’s Lemma) Suppose p

∈ N is a prime number; and sup-

pose a, b

∈ Z. Then

p

| ab =⇒ p | a or p | b.

Proof

I

Suppose p

| ab, p - a. We must show that p | b. Evidently

gcd(p, a) = 1.

Hence, by Corollary 1.1, there exist x, y

∈ Z such that

px + ay = 1.

Multiplying this equation by b,

pxb + aby = b.

But p

| pxb and p | aby (since p | ab). Hence

p

| b.

J

Theorem 1.3 Suppose n

∈ N, n > 0. Then n is expressible as a product of prime

numbers,

n = p

1

p

2

· · · p

r

,

and this expression is unique up to order.

Remark: We follow the convention that an empty product has value 1, just as an
empty sum has value 0. Thus the theorem holds for n = 1 as the product of no
primes.

background image

374

1–12

Proof

I

We prove existence by induction on n, the result begin trivial (by the

remark above) when n = 1. We know that n has at least one prime factor p, by
Lemma 1.1, say

n = pm.

Since m = n/p < n, we may apply our inductive hypothesis to m,

m = q

1

q

2

· · · q

s

.

Hence

n = pq

1

q

2

· · · q

s

.

Now suppose

n = p

1

p

2

· · · p

r

= m = q

1

q

2

· · · q

s

.

Since p

1

| n, it follows by repeated application of Euclid’s Lemma that

p

1

| q

j

for some j. But then it follows from the definition of a prime number that

p

1

= q

j

.

Again, we argue by induction on n. Since

n/p

1

= p

2

· · · p

r

= q

1

· · · ˆ

q

j

· · · q

s

(where the ‘hat’ indicates that the factor is omitted), and since n/p

1

< n, we

deduce that the factors p

2

, . . . , p

r

are the same as q

1

, . . . , ˆ

q

j

, . . . , q

s

, in some order.

Hence r = s, and the primes p

1

,

· · · , p

r

and q

1

, . . . , q

s

are the same in some order.

J

We can base another proof of Euclid’s Theorem (that there exist an infinity of

primes) on the fact that if there were only a finite number of primes there would
not be enough products to “go round”.

Thus suppose there were just m primes

p

1

, . . . , p

m

.

Let N

∈ N. By the Fundamental Theorem, each n ≤ N would be expressible in

the form

n = p

e

1

1

· · · p

e

m

m

.

(Actually, we are only using the existence part of the Fundamental Theorem; we
do not need the uniqueness part.)

For each i (1

≤ i ≤ m),

p

e

i

i

| n =⇒ p

e

i

i

≤ n

=

⇒ p

e

i

i

≤ N

=

⇒ 2

e

i

≤ N

=

⇒ e

i

≤ log

2

N.

background image

374

1–13

Thus there are at most log

2

N + 1 choices for each exponent e

i

, and so the number

of numbers n

≤ N expressible in this form is

≤ (log

2

N + 1)

m

.

So our hypothesis implies that

(log

2

N + 1)

m

≥ N

for all N .

But in fact, to the contrary,

X > (log

2

X + 1)

m

=

log X

log 2

+ 1

!

m

for all sufficiently large X. To see this, set X = e

x

. We have to show that

e

x

>

x

log 2

+ 1

!

m

.

Since

x

log 2

+ 1 < 2x

if x

≥ 3, it is sufficient to show that

e

x

> (2x)

m

for sufficiently large x. But

e

x

>

x

m+1

(m + 1)!

if x > 0, since the expression on the right is one of the terms in the power-series
expansion of e

x

. Thus the inequality holds if

x

m+1

(m + 1)!

> (2x)

m

,

ie if

x > 2

m

(m + 1)!.

We have shown therefore that m primes are insufficient to express all n

≤ N

if

N

≥ e

2

m

(m+1)!

.

Thus our hypothesis is untenable; and Euclid’s theorem is proved.

Our proof gives the bound

p

n

≤ e

2

m

(m+1)!

.

background image

374

1–14

which is even worse than the bound we derived from Euclid’s proof. (For it is
easy to see by induction that

(m + 1)! > e

m

for m

≥ 2. Thus our bound is worse than e

e

n

, compared with 2

2

n

by Euclid’s

method.)

We can improve the bound considerably by taking out the square factor in n.

Thus each number n

∈ N (n > 0) is uniquely expressible in the form

n = d

2

p

1

. . . p

r

,

where the primes p

1

, . . . , p

r

are distinct. In particular, if there are only m primes

then each n is expressible in the form

n = d

2

p

e

1

1

· · · p

e

m

m

,

where now each exponent e

i

is either 0 or 1.

Consider the numbers n

≤ N. Since

d

n

N ,

the number of numbers of the above form is

N 2

m

.

Thus we shall reach a contradiction when

N 2

m

≥ N,

ie

N

≤ 2

2m

.

This gives us the bound

p

n

≤ 2

2n

,

better than 2

2

n

, but still a long way from the truth.

1.5

The Fundamental Theorem, recast

We suppose throughout this section that A is an integral domain. (Recall that an
integral domain is a commutative ring with 1 having no zero divisors, ie if a, b

∈ A

then

ab = 0 =

⇒ a = 0 or b = 0.)

We want to examine whether or not the Fundamental Theorem holds in A —

we shall find that it holds in some commutative rings and not in others. But to
make sense of the question we need to re-cast our definition of a prime.

Looking back at Z, we see that we could have defined primality in two ways

(excluding p = 1 in both cases):

background image

374

1–15

1. p is prime if it has no proper factors, ie

p = ab =

⇒ a = 1 or b = 1.

2. p is prime if

p

| ab =⇒ p | a or p | b.

The two definitions are of course equivalent in the ring Z. However, in a

general ring the second definition is stronger: that is, an element satisfying it must
satisfy the first definition, but the converse is not necessarily true. We shall take
the second definition as our starting-point.

But first we must deal with one other point. In defining primality in Z we

actually restricted ourselves to the semi-ring N, defined by the order in Z:

N

=

{n ∈ Z : n ≥ 0}.

However, a general ring A has no natural order, and no such semi-ring, so we must
consider all elements a

∈ A.

In the case of Z this would mean considering

−p as a prime on the same

footing as p. But now, for the Fundamental Theorem to make sense, we would
have to regard the primes

±p as essentially the same.

The solution in the general ring is that to regard two primes as equivalent if

each is a multiple of the other, the two multiples necessarily being units.

Definition 1.5 An element

∈ A is said to be a unit if it is invertible, ie if there is

an element η

∈ A such that

η = 1.

We denote the set of units in A by A

×

.

For example,

Z

×

=

{±1}.

Proposition 1.5 The units in A form a multiplicative group A

×

.

Proof

I

This is immediate. Multiplication is associative, from the definition of a

ring; and η =

−1

is a unit, since it has inverse .

J

Now we can define primality.

Definition 1.6 Suppose a

∈ A is not a unit, and a 6= 0. Then

1. a is said to be irreducible if

a = bc =

⇒ b or c is a unit.

2. a is said to be prime if

a

| bc =⇒ a | b or p | b.

background image

374

1–16

Proposition 1.6 If a

∈ A is prime then it is irreducible.

Proof

I

Suppose

a = bc.

Then

a

| b or a | c.

We may suppose without loss of generality that a

| b. Then

a

| b, b | a =⇒ a = b,

where is a unit; and

a = bc = b =

⇒ c = .

J

Definition 1.7 The elements a, b

∈ A are said to be equivalent, written

a

∼ b,

if

b = a

for some unit .

In effect, the group of units A

×

acts on A and two elements are equivalent if

each is a transform of the other under this action.

Now we can re-state the Fundamental Theorem in terms which make sense in

any integral domain.

Definition 1.8 The integral domain A is said to be a unique factorisation domain
if each non-unit a

∈ A, a 6= 0 is expressible in the form

a = p

1

· · · p

r

,

where p

1

, . . . , p

r

are prime, and if this expression is unique up to order and equiv-

alence of primes.

In other words, if

a = q

1

· · · q

s

is another expression of the same form, then r = s and we can find a permutation
π of

{1, 2, . . . , r} and units

1

,

2

, . . . ,

r

such that

q

i

=

i

p

π(i)

for i = 1, 2, . . . , r.

Thus a unique factorisation domain (UFD) is an integral domain in which the

Fundamental Theorem of Arithmetic is valid.

background image

374

1–17

1.6

Principal ideals domains

Definition 1.9 The integral domain A is said to be a principal ideal domain if
every ideal
a

∈ A is principal, ie

a

=

hai = {ac : c ∈ A}

for some a

∈ A.

Example: By Proposition 1.3, Z is a principal ideal domain.

Our proof of the Fundamental Theorem can be divided into two steps — this

is clearer in the alternative version outlined in Section 1.3 — first we showed that
that Z is a principal ideal domain, and then we deduced from this that Z is a unique
factorisation domain.

As our next result shows this argument is generally available; it is the tech-

nique we shall apply to show that the Fundamental Theorem holds in a variety of
integral domains.

Proposition 1.7 A principal ideal domain is a unique factorisation domain.

Proof

I

Suppose A is a principal ideal domain.

Lemma 1.2 A non-unit a

∈ A, a 6= 0 is prime if and only if it is irreducible, ie

a = bc =

⇒ a is a unit or b is a unit.

Proof of Lemma

B

By Proposition 1.6, a prime is always irreducible.

The converse is in effect Euclid’s Lemma. Thus suppose

p

| ab but p - a.

Consider the ideal

hp, ai generated by p and a. By hypothesis this is principal, say

hp, ai = hdi.

Since p is irreducible,

d

| p =⇒ d = or d = p,

where is a unit. But

d = p, d

| a =⇒ p | a,

contrary to hypothesis. Thus d is a unit, ie

hp, ai = A.

In particular we can find u, v

∈ A such that

pu + av = 1.

background image

374

1–18

Multiplying by b,

pub + abv = b.

But now

p

| ab =⇒ p | b.

C

Now suppose a is neither a unit nor 0; and suppose that a is not expressible as

a product of primes. Then a is reducible, by the Lemma above: say

a = a

1

b

1

,

where a

1

, b

1

are non-units. One at least of a

1

, b

1

is not expressible as a product of

primes; we may assume without loss of generality that this is true of a

1

.

It follows by the same argument that

a

1

= a

2

b

2

,

where a

2

, b

2

are non-units, and a

2

is not expressible as a product of primes.

Continuing in this way,

a = a

1

b

1

, a

1

= a

2

b

2

, a

2

= a

3

b

3

, . . . .

Now consider the ideal

a

=

ha

1

, a

2

, a

3

, . . .

i.

By hypothesis this ideal is principal, say

a

=

hdi.

Since d

∈ a,

d

∈ ha

1

, . . . , a

r

i = ha

r

i

for some r. But then

a

r+1

∈ hdi = ha

r

i.

Thus

a

r

| a

r+1

, a

r+1

| a

r

=

⇒ a

r

= a

r+1

=

⇒ b

r+1

= ,

where is a unit, contrary to construction.

Thus the assumption that a is not expressible as a product of primes is unten-

able;

a = p

1

· · · p

r

.

To prove uniqueness, we argue by induction on r, where r the smallest number

such that a is expressible as a product of r primes.

Suppose

a = p

1

· · · p

r

= q

1

· · · q

s

.

Then

p

1

| q

1

· · · q

s

=

⇒ p

1

| q

j

background image

374

1–19

for some j. Since q

j

is irreducible, by Proposition 1.6, it follows that

q

j

= p

1

,

where is a unit.

We may suppose, after re-ordering the q’s that j = 1. Thus

p

1

∼ q

1

.

If r = 1 then

a = p

1

= p

1

q

2

· · · q

s

=

⇒ 1 = q

2

· · · q

s

.

If s > 1 this implies that q

2

, . . . , q

s

are all units, which is absurd. Hence s = 1,

and we are done.

If r > 1 then

q

1

= p

1

=

⇒ p

2

p

3

· · · p

r

= (q

2

)q

3

· · · q

s

(absorbing the unit into q

2

). The result now follows by our inductive hypothesis.

J

1.7

Polynomial rings

If A is a commutative ring (with 1) then we denote by A[x] the ring of polynomials

p(x) = a

n

x

n

+

· · · + a

0

(a

0

, . . . , a

n

∈ A).

Note that these polynomials should be regarded as formal expressions rather

than maps p : A

→ A; for if A is finite two different polynomials may well define

the same map.

We identify ainA with the constant polynomial f (x) = a. Thus

A

⊂ A[x].

Proposition 1.8 If A is an integral domain then so is A[x].

Proof

I

Suppose

f (x) = a

m

x

m

+

· · · + a

0

,

g(x) = b

n

x

n

+

· · · + b

0

,

where a

m

6= 0, b

n

6= 0. Then

f (x)g(x) = (a

m

b

n

)x

m+n

+

· · · + a

0

b

0

;

and the leading coefficient a

m

b

n

6= 0.

J

background image

374

1–20

Proposition 1.9 The units in A[x] are just the units of A:

(A[x])

×

= A

×

.

Proof

I

It is clear that a

∈ A is a unit (ie invertible) in A[x] if and only if it is a

unit in A.

On the other hand, no non-constant polynomial F (x)

∈ A[x] can be invertible,

since

deg F (x)G(x)

≥ deg F (x)

if G(x)

6= 0.

J

If A is a field then we can divide one polynomial by another, obtaining a

remainder with lower degree than the divisor. Thus degree plays the rˆole in k[x]
played by size in Z.

Proposition 1.10 Suppose k is a field; and suppose f (x), g(x)

∈ k[x], with

g(x)

6= 0. Then there exist unique polynomials q(x), r(x) ∈ k[x] such that

f (x) = g(x)q(x) + r(x),

where

deg r(x) < deg g(x).

Proof

I

We prove the existence of q(x), r(x) by induction on deg f (x).

Suppose

f (x) = a

m

x

m

+

· · · + a

0

,

g(x) = b

n

x

n

+

· · · + b

0

,

where a

m

6= 0, b

n

6= 0.

If m < n then we can take q(x) = 0, r(x) = f (x). We may suppose therefore

that m

≥ n. In that case, let

f

1

(x) = f (x)

− (a

m

/b

n

)x

m

−n

g(x).

Then

deg f

1

(x) < deg f (x).

Hence, by the inductive hypothesis,

f

1

(x) = g(x)q

1

(x) + r(x),

where

deg r(x) < deg g(x);

and then

f (x) = g(x)q(x) + r(x),

with

q(x) = (a

m

/b

n

)x

m

−n

+ q

1

(x).

background image

374

1–21

For uniqueness, suppose

f (x) = g(x)q

1

(x) + r

1

(x) = g(x)q

2

(x) + r

2

(x).

On subtraction,

g(x)q(x) = r(x),

where

q(x) = q

2

(x)

− q

1

(x),

r(x) = r

1

(x)

− r

2

(x).

But now, if q(x)

6= 0,

deg(g(x)q(x))

≥ deg g(x),

deg r(x) < deg g(x).

This is a contradiction. Hence

q(x) = 0,

ie

q

1

(x) = q

2

(),

r

1

(x) = r

2

().

J

Proposition 1.11 If k is a field then k[x] is a principal ideal domain.

Proof

I

As with Z we can prove this result in two ways: constructively, using the

Euclidean Algorithm; or non-constructively, using ideals. This time we take the
second approach.

Suppose

a

⊂ k[x]

is an ideal. If a = 0 the result is trivial; so we may assume that a

6= 0.

Let

d(x)

∈ a

be a polynomial in a of minimal degree. Then

a

=

hd(x)i.

For suppose f (x)

∈ a. Divide f(x) by d(x):

f (x) = d(x)q(x) + r(x),

where deg r(x) < deg d(x). Then

r(x) = f (x)

− d(x)q(x) ∈ a

since f (x), d(x)

∈ a. Hence, by the minimality of deg d(x),

r(x) = 0,

ie

f (x) = d(x)q(x).

J

By Proposition 1.7 this gives the result we really want.

background image

374

1–22

Corollary 1.2 If k is a field then k[x] is a unique factorisation domain.

Every non-zero polynomial f (x)

∈ k[x] is equivalent to a unique monic poly-

nomial, namely that obtained by dividing by its leading term. Thus each prime,
or irreducible, polynomial p(x)

∈ k[x] has a unique monic representative; and we

can restate the above Corollary in a simpler form.

Corollary 1.3 Each monic polynomial

f (x) = x

n

+ a

n

−1

x

n

−1

+

· · · + a

0

can be uniquely expressed (up to order) as a product of irreducible monic polyno-
mials:

f (x) = p

1

(x)

· · · p

r

(x).

1.8

Postscript

We end this Chapter with a result that we don’t really need, but which we have
come so close to it would be a pity to omit.

Suppose A is an integral domain. Let K be the field of fractions of A. (Recall

that K consists of the formal expressions

a

b

,

with a, b

∈ A, b 6= 0; where we set

a

b

=

c

d

if

ad = bc.

The map

a

7→

a

1

: A

→ K

is injective, allowing us to identify A with a subring of K.)

The canonical injection

A

⊂ K

evidently extends to an injection

A[x]

⊂ K[x].

Thus we can regard f (x)

∈ A[x] as a polynomial over K.

Proposition 1.12 If A is a unique factorisation domain then so is A[x].

Proof

I

First we must determine the primes in A[x].

Lemma 1.3 The element p

∈ A is prime in A[x] if and only if it is prime in A.

background image

374

1–23

Proof of Lemma

B

It is evident that

p prime in A[x] =

⇒ p prime in A.

Conversely, suppose p is prime in A; We must show that if F (x), G(x)

∈ A[x]

then

p

| F (x)G(x) =⇒ p | F (x) or p | G(x).

In other words,

p - F (x), p - G(x) =

⇒ p - F (x)G(x).

Suppose

F (x) = a

m

x

m

+

· · · + a

0

,

G(x) = b

n

x

n

+

· · · + b

0

;

and suppose

p - F (x),

p - G(x).

Let a

r

, b

s

be the highest coefficients of f (x), g(x) not divisible by p. Then the

coefficient of x

r+s

in f (x)g(x) is

a

0

b

r+s

+ a

1

b

r+s

−1

+

· · · + a

r

b

s

+

· · · + a

r+s

b

0

≡ a

r

b

s

mod p,

since all the terms except a

r

b

s

are divisible by p. Hence

p

| a

r

b

s

=

⇒ p mod a

r

or p mod b

s

,

contrary to hypothesis. In other words,

p - F (x)G(x).

C

Lemma 1.4 Suppose f (x)

∈ K[x]. Then f(x) is expressible in the form

f (x) = αF (x),

where α

∈ K and

F (x) = a

n

x

n

+

· · · + a

0

∈ A[x]

with

gcd(a

0

, . . . , a

n

) = 1;

and the expression is unique up to multiplication by a unit, ie if

f (x) = αF (x) = βG(x),

where G(x) has the same property then

G(x) = F (x),

α = β

for some unit

∈ A.

background image

374

1–24

Proof of Lemma

B

Suppose

f (x) = α

n

x

n

+

· · · + α

0

.

Let

α

i

=

a

i

b

i

,

where a

i

, b

i

∈ A; and let

b =

Y

b

i

.

Then

bf (x) = b

n

x

n

+

· · · + b

0

∈ A[x].

Now let

d = gcd(b

0

, . . . , b

n

).

Then

f (x) = (b/d)(c

n

x

n

+

· · · + c

0

)

is of the required form, since

gcd(c

0

, . . . , c

n

) = 1.

To prove uniqueness, suppose

f (x) = αF (x) = βG(x).

Then

G(x) = γF (x),

where γ = α/β.

In a unique factorisation domain A we can express any γ

∈ K in the form

γ =

a

b

,

with gcd(a, b) = 1, since we can divide a and b by any common factor.

Thus

aF (x) = bG(x).

Let p be a prime factor of b. Then

p

| aF (x) =⇒ p | F (x),

contrary to our hypothesis on the coefficients of F (x). Thus b has no prime factors,
ie b is a unit; and similarly a is a unit, and so γ is a unit.

C

Lemma 1.5 A non-constant polynomial

F (x) = a

n

x

n

+

· · · + a

0

∈ A[x]

is prime in A[x] if and only if

background image

374

1–25

1. F (x) is prime (ie irreducible) in K(x); and

2. gcd(a

0

, . . . , a

n

) = 1.

Proof of Lemma

B

Suppose F (x) is prime in A[x]. Then certainly

gcd(a

0

, . . . , a

n

) = 1,

otherwise F (x) would be reducible.

Suppose F (x) factors in K[x]; say

F (x) = g(x)h(x).

By Proposition 1.4,

g(x) = αG(x),

h(x) = βH(x),

where G(x), H(x) have no factors in A. Thus

F (x) = γG(x)H(x),

where γ

∈ K. Let γ = a/b, where a, b ∈ A and gcd(a, b) = 1. Then

bF (x) = aG(x)H(x).

Suppose p is a prime factor of b. Then

p

| G(x) or p | H(x),

neither of which is tenable. Hence b has no prime factors, ie b is a unit. But now

F (x) = ab

−1

G(x)H(x);

and so F (x) factors in A[x].

Conversely, suppose F (x) has the two given properties. We have to show that

F (x) is prime in A[x].

Suppose

F (x)

| G(x)H(x)

in A[x].

If F (x) is constant then

F (x) = a

∼ 1

by the second property, so

F (x)

| G(x) and F (x) | H(x).

We may suppose therefore that deg F (x)

≥ 1. Since K[x] is a unique factori-

sation domain (Corollary to Proposition 1.11),

F (x)

| G(x) or F (x) | H(x)

background image

374

1–26

in K[x]. We may suppose without loss of generality that

F (x)

| G(x)

in K[x], say

G(x) = F (x)h(x),

where h(x)

∈ K[x].

By Lemma 1.4 we can express h(x) in the form

h(x) = αH(x),

where the coefficients of H(x) are factor-free. Writing

α =

a

b

,

with gcd(a, b) = 1, we have

bG(x) = aF (x)H(x).

Suppose p is a prime factor of b. Then

p

| a or p | F (x) or p | H(x),

none of which is tenable. Hence b has no prime factors, ie b is a unit. Thus

F (x)

| G(x)

in A[x].

C

Now suppose

F (x) = a

n

x

n

+

· · · a

0

∈ A[x]

is not a unit in A[x].

If F (x) is constant, say F (x) = a, then the factorisation of a into primes in A

is a factorisation into primes in A[x], by Lemma 1.3. Thus we may assume that
deg F (x)

≥ 1.

Since K[x] is a unique factorisation domain (Corollary to Proposition 1.11),

F (x) can be factorised in K[x]:

F (x) = a

n

p

1

(x)

· · · p

s

(x),

where p

1

(x), . . . , p

s

(x) are irreducible monic polynomials in K[x]. By Lem-

mas 1.4 and 1.5 each p

i

(x) is expressible in the form

p

i

(x) = α

i

P

i

(x),

where P

i

(x) is prime in A[x].

Thus

F (x) = αP

1

(x)

· · · P

r

(x),

background image

374

1–27

where

α = a

n

α

1

· · · α

r

∈ K.

Let

α =

a

b

,

where gcd(a, b) = 1. Then

bF (x) = aP

1

(x)

· · · P

r

(x).

Let p be a prime factor of b. Then

p

| P

i

(x)

for some i, contrary to the definition of P

i

(x). Hence b has no prime factors, ie b

is a unit.

If a is a unit then we can absorb = a/b into P

1

(x):

F (x) = Q(x)P

2

(x)

· · · P

r

(x),

where Q(x) = (a/b)P

1

(x).

If a is not a unit then

ab

−1

= p

1

· · · p

s

,

where p

1

, . . . , p

s

are prime in A (and so in A[x] by Lemma 1.3); and

F (x) = p

1

· · · p

s

P

1

(x)

· · · P

r

(x),

as required.

Finally, to prove uniqueness, we may suppose that deg F (x)

≥ 1, since the

result is immediate if F (x) = a is constant.

Suppose

F (x) = p

1

· · · p

s

P

1

(x)

· · · P

r

(x) = q

1

· · · q

s

0

Q

1

(x)

· · · Q

r

0

(x).

Each P

i

(x), Q

j

(x) is prime in K[x] by Lemma 1.5. Since K[x] is a unique

factorisation domain (Corollary to Proposition 1.11) it follows that r = r

0

and

that after re-ordering,

Q

i

(x) = αP

i

(x),

where α

∈ K

×

. Let

α = a/b

with gcd(a, b) = 1. Then

aP

i

(x) = bQ

i

(x).

If p is a prime factor of b then

p

| bQ

i

(x) =

⇒ p | Q

i

(x),

background image

374

1–28

contrary to the definition of Q

i

(x). Thus b has no prime factors, and is therefore a

unit. Similarly a is a unit. Hence

Q

i

(x) =

i

P

i

(x),

where

i

∈ A is a unit.

Setting

=

Y

i

i

,

we have

p

1

· · · p

s

= q

1

· · · q

s

0

.

Since A is a unique factorisation domain, s = s

0

and after re-ordering,

q

j

= η

j

p

j

,

where η

j

∈ A is a unit.

We conclude that the prime factors of F (x) are unique up to order and equiv-

alence (multiplication by units), ie A[x] is a unique factorisation domain.

J

Example: There is unique factorisation in Z[x], since Z is a principal ideal domain
by Proposition 1.3 and so a unique factorisation domain by Proposition 1.7.

Note that Z[x] is not a principal ideal domain, since eg the ideal

a

=

h2, xi,

consisting of all polynomials

F (x) = a

n

x

n

+

· · · + a

0

with a

0

even, is not principals:

a

6= hG(x)i.

For if it were, its generator G(x) would have to be constant, since a contains

non-zero constants, and

deg G(x)H(x)

≥ deg G(x)

if H(x)

6= 0. But if G(x) = d then

a

∩ Z = h2i =⇒ d = ±2,

ie a consists of all polynomials with even coefficients. Since x

∈ a is not of this

form we conclude that a is not principal.

background image

Chapter 2

Number fields

2.1

Algebraic numbers

Definition 2.1 A number α

∈ C is said to be algebraic if it satisfies a polynomial

equation

f (x) = x

n

+ a

1

x

n

−1

+

· · · + a

n

= 0

with rational coefficients a

i

∈ Q.

For example,

2 and i/2 are algebraic.

A complex number is said to be transcendental if it is not algebraic. Both e

and π are transcendental. It is in general extremely difficult to prove a number
transcendental, and there are many open problems in this area, eg it is not known
if π

e

is transcendental.

Proposition 2.1 The algebraic numbers form a field ¯

Q

⊂ C.

Proof

I

If α satisfies the equation f (x) = 0 then

−α satisfies f(−x) = 0, while

1/α satisfies x

n

f (1/x) = 0 (where n is the degree of f (x)). It follows that

−α

and 1/α are both algebraic. Thus it is sufficient to show that if α, β are algebraic
then so are α + β, αβ.

Suppose α satisfies the equation

f (x)

≡ x

m

+ a

1

x

m

−1

+

· · · + a

m

= 0,

and β the equation

g(x)

≡ x

n

+ b

1

x

n

−1

+

· · · + b

n

= 0.

Consider the vector space

V =

i

β

j

: 0

≤ i < m, 0 ≤ j < ni

over Q spanned by the mn elements α

i

β

j

. Evidently

α + β, αβ

∈ V.

2–1

background image

374

2–2

But if θ

∈ V then the mn + 1 elements

1, θ, θ

2

, . . . , θ

mn

are necessarily linearly dependent (over Q), since dim V

≤ mn. In other words

θ satisfies a polynomial equation of degree

≤ mn. Thus each element θ ∈ V is

algebraic. In particular α + β and αβ are algebraic.

J

2.2

Minimal polynomials and conjugates

Recall that a polynomial p(x) is said to be monic if its leading coefficient — the
coefficient of the highest power of x — is 1:

p(x) = x

n

+ a

1

x

n

−1

+

· · · + a

n

.

Proposition 2.2 Each algebraic number α

∈ ¯

Q

satisfies a unique monic polyno-

mial m(x) of minimal degree.

Proof

I

Suppose α satisfies two monic polynomials m

1

(x), m

2

(x) of minimal

degree d. Then α also satisfies the polynomial

p(x) = m

1

(x)

− m

2

(x)

of degree < d; and if p(x)

6= 0 then we can make it monic by dividing by its

leading coefficient. This would contradict the minimality of m

1

(x). Hence

m

1

(x) = m

2

(x).

J

Definition 2.2 The monic polynomial m(x) satisfied by α

∈ ¯

Q

is called the min-

imal polynomial of α. The degree of the algebraic number α is the degree of its
minimal polynomial
m(x).

Proposition 2.3 The minimal polynomial m(x) of α

∈ ¯

Q

is irreducible.

Proof

I

Suppose to the contrary

m(x) = f (x)g(x)

where f (x), g(x) are of lower degrees than m(x). But then α must be a root of
one of f (x), g(x).

J

Definition 2.3 Two algebraic numbers α, β are said to be conjugate if they have
the same minimal polynomial.

Proposition 2.4 An algebraic number of degree d has just d conjugates.

background image

374

2–3

Proof

I

If the minimal poynomial of α is

m(x) = x

d

+ a

1

x

d

−1

+

· · · + a

d

,

then by definition the conjugates of α are the d roots α

1

= α, α

2

, . . . , α

d

of m(x):

m(x) = (x

− α

1

)(x

− α

2

)

· · · (x − α

d

).

These conjugates are distinct, since an irreducible polynomial m(x) over Q is

necessarily separable, ie it cannot have a repeated root. For if α were a repeated
root of m(x), ie

(x

− α)

2

| m(x)

then

(x

− α) | m

0

(x),

and so

(x

− α) | d(x) = gcd(m(x), m

0

(x)).

But

d(x)

| m(x)

and

1

≤ deg(d(x)) ≤ d − 1,

contradicting the irreducibility of m(x).

J

2.3

Algebraic number fields

Proposition 2.5 Every subfield K

⊂ C contains the rationals Q:

Q

⊂ K ⊂ C.

Proof

I

By definition, 1

∈ K. Hence

n = 1 +

· · · + 1 ∈ K

for each integer n > 0.

By definition, K is an additive subgroup of C. Hence

−1 ∈ K; and so

−n = (−1)n ∈ K

for each integer n > 0. Thus

Z

⊂ K.

Finally, since K is a field, each rational number

r =

n

d

∈ K

where n, d

∈ Z with d 6= 0.

J

We can consider any subfield K

⊂ C as a vector space over Q.

background image

374

2–4

Definition 2.4 An number field (or more precisely, an algebraic number field) is
a subfield
K

⊂ C which is of finite dimension as a vector space over Q. If

dim

Q

= d

then K is said to be a number field of degree d.

Proposition 2.6 There is a smallest number field K containing the algebraic
numbers
α

1

, . . . , α

r

.

Proof

I

Every intersection (finite or infinite) of subfields of C is a subfield of C;

so there is a smallest subfield K containing the given algebraic numbers, namely
the intersection of all subfields containing these numbers. We have to show that
this field is a number field, ie of finite dimension over Q.

Lemma 2.1 Suppose K

⊂ C is a finite-dimensional vector space over Q. Then

K is a number field if and only if it is closed under multiplication.

Proof of Lemma

B

If K is a number field then it is certainly closed under multi-

plication.

Conversely, if this is so then K is closed under addition and multiplication; so

we only have to show that it is closed under division by non-zero elements.

Suppose α

∈ V, α 6= 0. Consider the map

x

7→ αx : V → V.

This is a linear map over Q; and it is injective since

αx = 0 =

⇒ x = 0.

Since V is finite-dimensional it follows that the map is surjective; in particular,

αx = α

for some x

∈ V , ie

x = 1

∈ V.

Moreover

αx = 1

for some x

∈ V , ie α is invertible. Hence V is a field.

C

Now suppose α

i

is of degree d

i

(ie satisfies a polynomial equation of degree

d

i

over Q). Consider the vector space (over Q)

V =

i

1

1

· · · α

i

r

r

: 0

≤ i

1

< d

1

,

· · · , 0 ≤ i

r

< d

r

i.

It is readily verified that

α

i

V

⊂ V,

background image

374

2–5

and so

V V

⊂ V,

ie V is closed under multiplication.

It follows that V is a field; and since any field containing α

1

, . . . , α

r

must

contain these products, V is the smallest field containing α

1

, . . . , α

r

. Moreover V

is a number field since

dim

Q

V

≤ d

1

· · · d

r

.

J

Definition 2.5 We denote the smallest field containing α

1

, . . . , α

r

∈ C by Q(α

1

, . . . , α

r

).

Proposition 2.7 If α is an algebraic number of degree d then each element γ

Q

(α) is uniquely expressible in the form

a

0

+ a

1

α +

· · · + a

d

−1

α

d

−1

(a

0

, a

1

, . . . , a

d

−1

∈ Q).

Proof

I

It follows as in the proof of Proposition 2.6 that these elements do con-

stitute the field Q(α). And if two of the elements were equal then α would satisfy
an equation of degree < d, which could be made monic by dividing by the leading
coefficient.

J

A number field of the form K = Q(α), ie generated by a single algebraic

number α, is said to be simple. Our next result shows that, surprisingly, every
number field is simple. The proof is more subtle than might appear at first sight.

Proposition 2.8 Every number field K can be generated by a single algebraic
number:

K = Q(α).

Proof

I

It is evident that

K = Q(α

1

, . . . , α

r

);

for if we successively adjoin algebraic numbers

α

i+1

∈ K \ Q(α

1

, . . . , α

r

)

then

dim Q(α

1

) < dim Q(α

1

, α

2

) dim Q(α

1

, α

2

, α

3

) <

and so K must be attained after at most dim

Q

K adjunctions.

Thus it is suffient to prove the result when r = 2, ie to show that, for any two

algebraic numbers α, β,

Q

(α, β) = Q(γ).

Let p(x) be the minimal polynomial of α, and q(x) the minimal polynomial

of β. Suppose α

1

= α, . . . , α

m

are the conjugates of α and β

1

= β, . . . , β

n

the

conjugates of β. Let

γ = α + aβ,

background image

374

2–6

where a

∈ Q is chosen so that the mn numbers

α

i

+ aβ

j

are all distinct. This is certainly possible, since

α

i

+ aβ

j

= α

i

0

+ aβ

j

0

⇐⇒ a =

α

i

0

− α

i

β

j

− β

j

0

.

Thus a has to avoid at most mn(mn

− 1)/2 values.

Since

α = γ

− aβ,

and

p(α) = 0,

β satisfies the equation

p(γ

− ax) = 0.

This is a polynomial equation over the field k = Q(γ).

But β also satisfies the equation

q(x) = 0.

It follows that β satisfies the equation

d(x) = gcd(p(γ

− ax), q(x)) = 0.

Now

(x

− β) | d(x)

since β is a root of both polynomials. Also, since

d(x)

| q(x) = (x − β

1

)

· · · (x − β

n

),

d(x) must be the product of certain of the factors (x

− β

j

). Suppose (x

− β

j

) is

one such factor. Then β

j

is a root of p(γ

− ax), ie

p(γ

− aβ

j

) = 0.

Thus

γ

− aβ

j

= α

i

for some i. Hence

γ = α

i

+ aβ

j

.

But this implies that i = 1, j = 1, since we chose a so that the elements

α

i

+ aβ

j

were all distinct.

background image

374

2–7

Thus

d(x) = (x

− β).

But if u(x), v(x)

∈ k[x] then we can compute gcd(u(x), v(x)) by the eu-

clidean algorithm without leaving the field k, ie

u(x), v(x)

∈ k[x] =⇒ gcd(u(x), v(x)) ∈ k[x].

In particular, in our case

x

− β ∈ k = Q(γ).

But this means that

β

∈ Q(γ);

and so also

α = γ

− aβ ∈ Q(γ).

Thus

α, β

∈ Q(γ) =⇒ Q(α, β) ⊂ Q(γ) ⊂ Q(α, β).

Hence

Q

(α, β) = Q(γ).

J

2.4

Algebraic integers

Definition 2.6 A number α

∈ C is said to be an algebraic integer if it satisfies a

polynomial equation

f (x) = x

n

+ a

1

x

n

−1

+

· · · + a

n

= 0

with integral coefficients a

i

∈ Z. We denote the set of algebraic integers by ¯

Z

.

Proposition 2.9 The algebraic integers form a ring ¯

Z

with

Z

⊂ ¯

Z

⊂ ¯

Q

.

Proof

I

Evidently

Z

⊂ ¯

Z

,

since n

∈ Z satisfies the equation

x

− n = 0.

We have to show that

α, β

∈ ¯

Z

=

⇒ α + β, αβ ∈ ¯

Z

.

background image

374

2–8

Lemma 2.2 The number α

∈ C is an algebraic integer if and only if there exists

a finitely-generated (but non-zero) additive subgroup S

⊂ C such that

αS

⊂ S.

Proof of Lemma

B

Suppose α

∈ ¯

Z

; and suppose the minimal polynomial of α is

m(x) = x

d

+ a

1

x

d

−1

+

· · · + a

d

,

where a

1

, . . . , a

d

∈ Z. Let S be the abelian group generated by 1, α, . . . , α

d

−1

:

S =

h1, α, . . . , α

d

−1

i.

Then it is readily verified that

αS

⊂ S.

Conversely, suppose S is such a subgroup.

C

If α is a root of the monic polynomial f (x) then

−α is a root of the monic

polynomial f (

−x). It follows that if α is an algebraic integer then so is −α. Thus

it is sufficient to show that if α, β are algebraic integers then so are α + β, αβ.

Suppose α satisfies the equation

f (x)

≡ x

m

+ a

1

x

m

−1

+

· · · + a

m

= 0

(a

1

, . . . , a

m

∈ Z),

and β the equation

g(x)

≡ x

n

+ b

1

x

n

−1

+

· · · + b

n

= 0

(b

1

, . . . , b

n

∈ Z).

Consider the abelian group (or Z-module)

M =

i

β

j

: 0

≤ i < m, 0 ≤ j < ni

generated by the mn elements α

i

β

j

. Evidently

α + β, αβ

∈ V.

As a finitely-generated torsion-free abelian group, M is isomorphic to Z

d

for

some d. Moreover M is noetherian, ie every increasing sequence of subgroups of
M is stationary: if

S

1

⊂ S

2

⊂ S

3

· · · ⊂ M

then for some N ,

S

N

= S

N +1

= S

N +2

=

· · · .

Suppose θ

∈ M. Consider the increasing sequence of subgroups

h1i ⊂ h1, θi ⊂ h1, θ, θ

2

i ⊂ · · · .

This sequence must become stationary; that is to say, for some N

θ

N

∈ h1, θ, . . . , θ

N

−1

i.

In other words, θ satisfies an equation of the form

θ

N

= a

1

θ

N

−1

+ a

2

θ

N

−2

+

· · · .

Thus every θ

∈ M is an algebraic integer. In particular α+β and αβ are algebraic

integers.

J

background image

374

2–9

Proposition 2.10 A rational number c

∈ Q is an algebraic integer if and only if

it is a rational integer:

¯

Z

∩ Q = Z.

Proof

I

Suppose c = m/n, where gcd(m, n) = 1; and suppose c satisfies the

equation

x

d

+ a

1

x

d

−1

+

· · · + a

d

= 0

(a

i

∈ Z).

Then

m

d

+ a

1

m

d

−1

n +

· · · + a

d

n

d

= 0.

Since n divides every term after the first, it follows that n

| m

d

. But that is

incompatible with gcd(m, n) = 1, unless n = 1, ie c

∈ Z.

J

Proposition 2.11 Every algebraic number α is expressible in the form

α =

β

n

,

where β is an algebraic integer, and n

∈ Z.

Proof

I

Let the minimal polynomial of α be

m(x) = x

d

+ a

1

x

d

−1

+

· · · + a

d

,

where a

1

, . . . , a

d

∈ Q. Let the lcm of the denominators of the a

i

be n. Then

b

i

= na

i

∈ Z (1 ≤ i ≤ d).

Now α satisfies the equation

nx

d

+ b

1

x

d

−1

+

· · · + b

d

= 0.

It follows that

β = nα

satisfies the equation

x

d

+ b

1

x

d

−1

+ (nb

2

)x

d

−2

+

· · · + (n

d

−1

b

d

= 0.

Thus β is an integer, as required.

J

The following result goes in the opposite direction.

Proposition 2.12 Suppose α is an algebraic integer. Then we can find an alge-
braic integer
β

6= 0 such that

αβ

∈ Z.

background image

374

2–10

Proof

I

Let the minimal polynomial of α be

m(x) = x

d

+ a

1

x

d

−1

+

· · · + a

d

,

where a

1

, . . . , a

d

∈ Z. Recall that the conjugates of α,

α

1

= α, . . . , α

d

are the roots of the minimal equation.

Each of these conjugates is an algebraic integer, since its minimal equation

m(x) has integer coefficients. Hence

β = α

2

· · · α

d

is an algebraic integer; and

αβ = α

1

α

2

· · · α

d

=

±a

d

∈ Z.

J

2.5

Units

Definition 2.7 A number α

∈ C is said to be a unit if both α and 1/α are alge-

braic integers.

Any root of unity, ie any number satisfying x

n

= 1 for some n, is a unit.

But these are not the only units; for example,

2

− 1 is a unit.

The units form a multiplicative subgroup of ¯

Q

×

.

2.6

The Integral Basis Theorem

Proposition 2.13 Suppose A is a number ring. Then we can find γ

1

, . . . , γ

d

∈ A

such that each α

∈ A is uniquely expressible in the form

α = c

1

γ

1

+ c

d

γ

d

with c

1

, . . . , c

d

∈ Z.

In other words, as an additive group

A ∼

= Z

d

.

We may say that γ

1

, . . . , γ

d

is a Z-basis for A.

Proof

I

Suppose A is the ring of integers in the number field K. By Proposi-

tion 2.8,

K = Q(α).

background image

374

2–11

By Proposition 2.12,

α =

β

m

,

where β

∈ ¯

Z

, m

∈ Z. Since

Q

(β) = Q(α),

we may suppose that α is an integer.

Let

m(x) = x

d

+ a

1

x

d

−1

+

· · · + a

d

be the minimal polynomial of α; and let

α

1

= α, . . . , α

d

be the roots of this polynomial, ie the conjugates of α.

Note that these conjugates satisfy exactly the same set of polynomials over Q;

for

p(α) = 0

⇐⇒ m(x) | p(x) ⇐⇒ p(α

i

) = 0.

Now suppose β

∈ A. Then

β = b

0

+ b

1

α +

· · · b

d

−1

α

d

−1

,

where b

0

, . . . , b

d

−1

∈ Q, say

β = f (α)

with f (x)

∈ Q[x].

Let

β

i

= b

0

+ b

1

α

i

+

· · · b

d

−1

α

d

−1

i

for i = 1, . . . , d.

Each β

i

satisfies the same set of polynomials over Q as β. for

p(β) = 0

⇐⇒ p(f(α)) = 0 ⇐⇒ p(f(α

i

)) = 0

⇐⇒ p(β

i

) = 0.

In particular, each β

i

has the same minimal polynomial as β, and so each β

i

is an

integer.

We may regard the formulae for the β

i

as linear equations for the coefficients

b

0

, . . . , b

d

−1

:

b

0

+ α

1

b

1

+

· · · α

d

−1

b

d

−1

= β

1

,

. . .

b

0

+ α

d

b

1

+

· · · α

d

−1

d

b

d

−1

= β

d

.

We can write this as a matrix equation

D



b

0

..

.

b

d

−1



=



β

1

..

.

β

d



background image

374

2–12

where D is the matrix

D =



1

α

1

. . .

α

d

−1

1

..

.

. . .

. . .

..

.

1

α

d

. . .

α

d

−1

d

.



By a familiar argument,

det



1

x

1

. . .

x

d

−1

1

..

.

. . .

. . .

..

.

1

x

d

. . .

x

d

−1

d



=

Y

i<j

(x

i

− x

j

).

(The determinant vanishes whenever x

i

= x

j

since then two rows are equal.

Hence (x

i

− x

j

) is a factor for each pair i, j; from which the result follows on

comparing degrees and leading coefficients.)

Thus

det D =

Y

i<j

i

− α

j

).

In particular, det D is an integer.

On solving the equations for b

0

, . . . , b

d

−1

by Cramer’s rule, we deduce that

b

i

=

β

i

det D

,

where β

i

is a co-factor of the matrix D, and so a polynomial in α

1

, . . . , α

d

with

coefficients in Z, and therefore an algebraic integer.

By Proposition 2.12, we can find an integer δ such that

δ det D = n

∈ Z,

where we may suppose that n > 0. Thus each b

i

is expressible in the form

b

i

=

γ

i

n

,

where

γ

i

∈ ¯

Z

∩ Q = Z.

In other words, each β

∈ A is expressible in the form

β = c

o

δ

0

+

· · · + c

d

−1

δ

d

−1

,

where

δ

i

=

α

i

n

and

c

i

∈ Z (0 ≤ i < d).

The elements

c

o

δ

0

+

· · · + c

d

−1

δ

d

−1

(c

i

∈ Z)

form a finitely-generated and torsion-free abelian group C, of rank d; and A is
a subgroup of C of finite index. We need the following standard result from the
theory of finitely-generated abelian groups.

background image

374

2–13

Lemma 2.3 If

S

⊂ Z

d

is a subgroup of finite index then

S ∼

= Z

d

Proof of Lemma

B

We have to construct a Z-basis for S. We argue by induction

on d.

Choose an element

e = (e

1

, . . . , e

d

)

∈ S

with least positive last coordinate e

d

. Suppose

s = (s

1

, . . . , s

d

)

∈ S.

Then

s

d

= qe,

or we could find an element of S with smaller last coordinate. Thus

s

− qe = (t

1

, . . . , t

d

−1

, 0).

Hence

S = Ze

⊕ T,

where

T = S

∩ Z

d

−1

(identifying Z

d

−1

with the subgroup of Z

d

formed by the d-tuples with last coor-

dinate 0).

The result follows on applying the inductive hypothesis to T .

C

The Proposition follows on applying the Lemma to

A

⊂ C ∼

= Z

d

.

J

2.7

Unique factorisation in number rings

As we saw in Chapter 1, a principal ideal domain is a unique factorisation domain.
The converse is not true; there is unique factorisation in Z[x], but the ideal

h2, xi

is not principal. Our main aim in this Section is to show that the converse does
hold for number rings A:

A principal ideal domain

⇐⇒ A unique factorisation domain.

We suppose throughout the Section that A is a number ring, ie the ring of

integers in a number field K.

background image

374

2–14

Proposition 2.14 Suppose a

⊂ A is a non-zero ideal. Then the quotient-ring

A/a

is finite.

Proof

I

Take α

∈ a, α 6= 0. By Proposition 1.8, we can find β ∈ A, β 6= 0 such

that

a = αβ

∈ Z.

We may suppose that a > 0. Then

hai ⊂ hαi ⊂ a.

Thus

α

≡ β mod a =⇒ α ≡ β mod a.

By Proposition 2.13, A has an integral basis γ

1

, . . . , γ

d

, ie each α

∈ A is

(uniquely) expressible in the form

α = c

1

γ

1

+

· · · + c

d

γ

d

with c

1

, . . . , c

d

∈ Z. It follows that α is congruent moda to one of the numbers

r

1

γ

1

+ r

d

γ

d

(0

≤ r

i

< a).

Thus

kA/haik = a

d

.

Hence

kA/ak ≤ a

d

.

J

Proposition 2.15 The number ring A is a unique factorisation domain if and only
if it is a principal ideal domain.

Proof

I

We know from Chapter 1 that

A principal ideal domain =

⇒ A unique factorisation domain.

We have to proce the converse.

Let us suppose therefore that the number ring A is a unique factorisation do-

main.

Lemma 2.4 Suppose

α = π

e

1

1

· · · π

e

r

r

,

β =

0

π

f

1

1

· · · π

f

r

r

.

Let

δ = π

min(e

1

,f

1

)

1

· · · π

min(e

r

,f

r

)

r

.

Then

δ = gcd(α, β)

in the sense that

δ

| α, δ | β and δ

0

| α, δ | β =⇒ δ

0

| δ.

background image

374

2–15

Proof of Lemma

B

This follows at once from unique factorisation.

C

Lemma 2.5 If

β

1

≡ β

2

mod α

then

gcd(α, β

1

) = gcd(α, β

2

).

Proof of Lemma

B

It is readily verified that if

β

1

= β

2

+ αγ

then

δ

| α, β

1

⇐⇒ δ | α, β

2

.

C

We say that α, β are coprime if

gcd(α, β) = 1.

It follows from the Lemma that we may speak of a congruence class ¯

β mod α

being coprime to α.

Lemma 2.6 The congruence classes modα coprime to α form a multiplicative
group

(A/

hαi)

×

.

Proof of Lemma

B

We have

gcd(α, β

1

β

2

) = 1

⇐⇒ gcd(α, β

1

) = 1, gcd(α, β

2

) = 1.

Thus (A/

hαi)

×

is closed under multiplication; and if β is coprime to α then the

map

¯

γ

7→ ¯

β ¯

γ : (A/

hαi)

×

→ (A/hαi)

×

is injective, and so surjective since A/

hαi is finite. Hence (A/hαi)

×

is a group.

C

Lemma 2.7 Suppose

gcd(α, β) = δ.

Then we can find u, v

∈ A such that

αu + βv = δ.

background image

374

2–16

Proof of Lemma

B

We may suppose, on dividing by δ, that

gcd(α, β) = 1,

and so

¯

β

∈ (A/hαi)

×

.

Since this group is finite,

¯

β

n

= 1

for some n > 0. In other words,

β

n

≡ 1 mod α,

ie

β

n

= 1 + αγ,

ie

αu + βv = 1

with u =

−γ, v = β

n

−1

.

C

We can extend the definition of gcd to any set (finite or infinite) of numbers

α

i

∈ A (i ∈ I).

and by repeated application of the last Lemma we can find β

i

(all but a finite

number equal to 0) such that

X

i

∈I

α

i

β

i

= gcd

i

∈I

i

).

Applying this to the ideal a, let

δ = gcd

α

∈a

(α).

Then

δ =

X

α

i

β

i

∈ a;

and so

a

=

hδi.

J

background image

Chapter 3

Quadratic Number Fields

3.1

The fields Q(

m)

Definition 3.1 A quadratic field is a number field of degree 2.

Recall that this means the field k has dimension 2 as a vector space over Q:

dim

Q

k = 2.

Definition 3.2 The integer m

∈ Z is said to be square-free if

m = r

2

s =

⇒ r = ±1.

Thus

±1, ±2, ±3, ±5, ±6, ±7, ±10, ±11, ±13, . . .

are square-free.

Proposition 3.1 Each quadratic field is of the form Q(

m) for a unique square-

free integer m

6= 1.

Recall that Q(

m) consists of the numbers

x + y

m

(x, y

∈ Q).

Proof

I

Suppose k is a quadratic field. Let α

∈ k \ Q. Then α

2

, α, 1 are linearly

dependent over Q, since dim

Q

k = 2. In other words, α satisfies a quadratic

equation

a

0

α

2

+ a

1

α + a

2

= 0

with a

0

, a

1

, a

2

∈ Q. We may assume that a

0

, a

1

, a

2

∈ Z. Then

α =

−a

1

+

q

a

2

1

− 4a

0

a

2

2a

0

3–1

background image

374

3–2

Thus

q

a

2

1

− 4a

0

a

2

= 2a

0

α + a

1

∈ k.

Let

a

2
1

− 4a

0

a

2

= r

2

m

where m is square-free. Then

m =

1

r

q

a

2

1

− 4a

0

a

2

∈ k.

Thus

Q

⊂ Q(

m)

⊂ k.

Since dim

Q

k = 2,

k = Q(

m).

To see that different square-free integers m

1

, m

2

give rise to different quadratic

fields, suppose

m

1

∈ Q(

m

2

),

say

m

1

= x + y

m

2

(x, y

∈ Q)

Squaring,

m

1

= x

2

+ m

2

y

2

+ 2xy

m

2

.

Thus either x = 0 or y = 0 or

m

2

∈ Q,

all of which are absurd.

J

When we speak of the quadratic field Q(

m) it is understood that m is a

square-free integer

6= 1.

Definition 3.3 The quadratic field Q(

m) is said to be real if m > 0, and imag-

inary if m < 0.

This is a natural definition since it means that Q(

m) is real if and only if

Q

(

m)

⊂ R.

3.2

Conjugates and norms

Proposition 3.2 The map

x + y

m

7→ x − y

m

is an automorphism of Q(

m); and it is the only such automorphism apart from

the identity map.

background image

374

3–3

Proof

I

The map clearly preserves addition. It also preserves multiplication, since

(x + y

m)(u + v

m = (xu + yvm) + (xv + yu)

m,

and so

(x

− y

m)(u

− v

m = (xu + yvm)

− (xv + yu)

m.

Since the map is evidently bijective, it is an automorphism.

Conversely, if θ is an automorphism of Q(

m) then θ preserves the elements

of Q; in fact if α

∈ Q(

m) then

θ(α) = α

⇐⇒ α ∈ Q.

Thus

θ(

m)

2

= θ(m) = m =

⇒ θ(

m) =

±

m,

giving the identity automorphism and the automorphism above.

J

Definition 3.4 If

α = x + y

m

(x, y

∈ Q)

then we write

¯

α = x

− y

m

(x, y

∈ Q)

and we call ¯

α the conjugate of α.

Note that if Q(

m) is imaginary (ie m < 0) then the conjugate ¯

α coincides

with the usual complex conjugate.

Definition 3.5 We define the norm

kαk of α ∈ Q(

m) by

kαk = α¯

α.

Thus if

α = x + y

m

(x, y

∈ Q)

then

kαk = (x + y

m)(x

− y

m) = x

2

− my

2

.

Proposition 3.3

1.

kαk ∈ Q;

2.

k(kα = 0 ⇐⇒ α = 0;

3.

kαβk = kαkkβk;

4. If a

∈ Q then kak = a

2

;

5. If m < 0 then

kαk ≥ 0.

Proof

I

All is clear except perhaps the third part, where

kαβk = (αβ)(αβ)

= (αβ)( ¯

α ¯

β)

= (α ¯

α)(β ¯

β)

=

kαkkβk.

J

background image

374

3–4

3.3

Integers

Proposition 3.4 Suppose k = Q(

m), where m

6= 1 is square-free.

1. If m

6≡ 1 mod 4 then the integers in k are the numbers

a + b

m,

where a, b

∈ Z.

2. If m

≡ 1 mod 4 then the integers in k are the numbers

a

2

+

b

2

m,

where a, b

∈ Z and

a

≡ b mod 2,

ie a, b are either both even or both odd.

Proof

I

Suppose

α = a + b

m

( b

∈ Q)

is an integer. Recall that an algebraic number α is an integer if and only if its
minimal polynomial has integer coefficients. If y = 0 the minimal polynomial of
α is x

− a. Thus α = a is in integer if and only if a ∈ Z (as we know of course

since ¯

Z

∩ Q = Z).

If y

6= 0 then the minimal polynomial of α is

(x

− a)

2

− mb

2

= x

2

− 2ax + (a

2

− mb

2

).

Thus α is an integer if and only if

2a

∈ Z and a

2

− mb

2

∈ Z.

Suppose 2a = A, ie

a =

A

2

.

Then

4a

2

∈ Z, a

2

− mb

2

∈ Z =⇒ 4mb

2

∈ Z

=

⇒ 4b

2

∈ Z

=

⇒ 2b ∈ Z

since m is square-free. Thus

b =

B

2

,

where B

∈ Z.

background image

374

3–5

Now

a

2

− mb

2

=

A

2

− mB

2

4

∈ Z,

ie

A

2

− mB

2

≡ 0 mod 4.

If A is even then

2

| A =⇒ 4 | A

2

=

⇒ 4 | mB

2

=

⇒ 2 | B

2

=

⇒ 2 | B;

and similarly

2

| B =⇒ 4 | B

2

=

⇒ 4 | A

2

=

⇒ 2 | A.

Thus A, B are either both even, in which case a, b

∈ Z, or both odd, in which case

A

2

, B

2

≡ 1 mod 4,

so that

1

− m ≡ 0 mod 4,

ie

m

≡ 1 mod 4.

Conversely if m

≡ 1 mod 4 then

A, B odd =

⇒ A

2

− mB

2

≡ 0 mod 4

=

⇒ a

2

− mb

2

∈ Z.

J

It is sometimes convenient to express the result in the following form.

Corollary 3.1 Let

ω =

m

if m

6≡ 1 mod 4,

1+

m

2

if m

≡ 1 mod 4.

Then the integers in Q(

m) form the ring Z[ω].

Examples:

1. The integers in the gaussian field Q(i) are the gaussian integers

a + bi

(a, b

∈ Z)

background image

374

3–6

2. The integers in Q(

2) are the numbers

a + b

2

(a, b

∈ Z).

3. The integers in Q(

−3) are the numbers

a + bω

(a, b

∈ Z)

where

ω =

1 +

−3

2

.

Proposition 3.5 If α

∈ Q(

m) is an integer then

kαk ∈ Z.

Proof

I

If α is an integer then so is its conjugate ¯

α (since α, ¯

α satisfy the same

polynomial equations over Q). Hence

kαk ∈ ¯

Z

∩ Q = Z.

J

3.4

Units

Proposition 3.6 An integer

∈ Q(

m) is a unit if and only if

kk = ±1.

Proof

I

Suppose is a unit, say

η = 1.

Then

kkkηk = k1k = 1.

Hence

kk = ±1.

Conversely, suppose

kk = ±1,

ie

¯

=

±1.

Then

−1

=

±¯

is an integer, ie is a unit.

J

background image

374

3–7

Proposition 3.7 An imaginary quadratic number field contains only a finite num-
ber of units.

1. The units in Q(i) are

±1, ±i;

2. The units in Q(

−3) are ±1, ±ω, ±ω

2

, where ω = (1 +

−3)/2.

3. In all other cases the imaginary quadratic number field Q(

m) (where

m < 0) has just two units,

±1.

Proof

I

We know of course that

±1 are always units.

Suppose

= a + b

m

is a unit. Then

N )) = a

2

+ (

−m)b

2

= 1

by Proposition 3.6. In particular

(

−m)b

2

≤ 1.

If m

≡ 3 mod 4 then a, b ∈ Z; and so b = 0 unless m = −1 in which case

b =

±1 is a solution, giving a = 0, ie = ±i.

If m

≡ 1 mod 4 then b may be a half-integer, ie b = B/2, and

(

−m)b

2

= (

−m)B

2

/4 > 1

if B

6= 0, unless m = −3 and B = ±1, in which case A = ±1. Thus we get four

additional units in Q(

−3), namely ±ω, ±ω

2

.

J

Proposition 3.8 Every real quadratic number field Q(

m) (where m > 0) con-

tains an infinity of units. More precisely, there is a unique unit η > 1 such that the
units are the numbers

±η

n

(n

∈ Z)

Proof

I

The following exercise in the pigeon-hole principle is due to Kronecker.

Lemma 3.1 Suppose α

∈ R. There are an infinity of integers m, n with m > 0

such that

|mα − n| <

1

n

.

Proof of Lemma

B

Let

{x} denote the fractional part of x ∈ R. Thus

{x} = x − [x],

where [x] is the integer part of x.

Suppose N is a positive integer. Let us divide [0, 1) into N equal parts:

[0, 1/N ), [1/N, 2/N ), . . . , [(N

− 1)/N, 1).

background image

374

3–8

Consider how the N + 1 fractional parts

{0}, {α}, {2α}, . . . , {Nα}

fall into these N divisions.

Two of the fractional parts — say

{rα} and {sα}, where r < s — must fall

into the same division. But then

|{sα} − {rα}| < 1/N,

ie

|(sα − [sα]) − (rα − [rα])| < N.

Let

m = s

− r, n = [sα] − [rα].

Then

|mα − n| < 1/N ≤ 1/m.

C

Lemma 3.2 There are an infinity of a, b

∈ Z such that

|a

2

− b

2

m

| < 2

m + 1.

Proof of Lemma

B

We apply Kronecker’s Lemma above with α =

m. There are

an infinity of integers a, b > 0 such that

|a − b

m

| < 1/b.

But then

a < b

m + 1,

and so

a + b

m < 2b

m + 1

Hence

|a

2

− b

2

m

| = (a + b

m)

|a − b

m

|

< (2b

m + 1)/b

≤ 2

m + 1.

C

It follows from this lemma that there are an infinity of integer solutions of

a

2

− b

2

m = d

for some

d < 2

m + 1.

But then there must be an infinity of these solutions (a, b) with the same re-

mainders modd.

background image

374

3–9

Lemma 3.3 Suppose

α

1

= a

1

+ b

1

m, α

2

= a

2

+ b

2

m,

where

a

2
1

− b

2
1

= d = a

2
2

− b

2
2

and

a

1

≡ a

2

mod d,

b

1

≡ b

2

mod d.

Then

α

1

α

2

is an algebraic integer.

Proof of Lemma

B

Suppose

a

2

= a

1

+ mr, b

2

= b

1

+ ms.

Then

α

2

= α

1

+ dβ,

where

β = r + s

m.

Hence

α

1

α

2

=

α

1

¯

α

2

α

2

¯

α

2

=

α

1

¯

α

2

d

=

α

1

( ¯

α

1

+ d ¯

β)

d

=

α

1

¯

α

1

d

+ ¯

β

=

d

d

+ β

= 1 + β,

which is an integer.

C

Now suppose (a

1

, b

1

), (a

2

, b

2

) are two such solutions. Then

=

α

1

α

2

is an integer, and

kk =

1

k

2

k

=

d

d

= 1.

Hence is a unit, by Proposition 3.6.

background image

374

3–10

Since there are an infinity of integers α satisfying these conditions, we obtain

an infinity of units if we fix α

1

and let α

2

vary. In particular there must be a unit

6= ±1.

Just one of the four units

±, ±

−1

must lie in the range (1,

∞). (The others are distributes one each in the ranges

(

−∞, −1), (−1, 0) and (0, 1).)

Suppose then that

= a + b

m > 1.

Then

|

−1

| < 1,

and so

¯

=

±

−1

∈ (−1, 1),

ie

−1 < a − b

m < 1.

Adding these two inequalities,

0 < 2a,

ie

a > 0.

On the other hand,

> ¯

=

⇒ b > 0.

It follows that there can only be a finite number of units in any range

1 <

≤ c.

In particular, if > 1 is a unit, then there is a smallest unit η in the range

1 < η

≤ .

Evidently η is the least unit in the range

1 < η.

Now suppose is a unit

6= ±1. As we observed, one of the four units ±, ±

−1

must lie in the range (1,

∞). We can take this in place of , ie we may assume that

> 1.

background image

374

3–11

Since η

n

→ ∞,

η

r

≤ < η

r+1

for some r

≥ 1. Hence

1

≤ η

−r

< η.

Since η is the smallest unit > 1, this implies that

η

−1

= 1,

ie

= η

r

.

J

3.5

Unique factorisation

Suppose A is an integral domain. Recall that if A is a principal ideal domain, ie
each ideal

A ⊂ A can be generated by a single element a,

a

=

hai,

then A is a unique factorisation domain, ie each a

∈ A is uniquely expressible —

up to order, and equivalence of primes — in the form

a = π

e

1

1

· · · π

e

r

r

,

where is a unit, and π

1

, . . . , π

r

are inequivalent primes.

We also showed that if A is the ring of integers in an algebraic number field k

then the converse is also true, ie

A principal ideal domain

⇐⇒ A unique factorisation domain .

Proposition 3.9 The ring of integers Z[ω] in the quadratic field Q(

m is a prin-

cipal ideal domain (and so a unique factorisation domain) if

m =

−11, −7, −3, −2, −1, 2, 3, 5, 13.

Proof

I

We take

|kαk|

as a measure of the size of α

∈ Z[ω].

Lemma 3.4 Suppose α, β

∈ Z[ω[, with β 6= 0. Then there exist γ, ρ ∈ Z[ω] such

that

α = βγ + ρ

with

|kρk| < |kβk|.

In other words, we can divide α by β, and get a remainder ρ smaller than β.

background image

374

3–12

Proof of Lemma

B

Let

α

β

= x + y

m

where x, y

∈ Q.

Suppose first that m

6≡ 1 mod 4. We can find integers a, b such that

|x − a|, |y − b| ≤

1

2

.

Let

γ = a + b

m.

Then γ

∈ Z[ω]; and

α

β

− γ = (x − a) + (y − b)

m.

Thus

k

α

β

− γk = (x − a)

2

− m(y − b)

2

.

If now m < 0 then

0

≤ k

α

β

− γk ≤

1 + m

4

,

yielding

|k

α

β

− γk| < 1

if m =

−2 or − 1; while if m > 0 then

m

4

≤ k

α

β

− γk ≤

1

4

,

yielding

|k

α

β

− γk| < 1

if m = 2 or 3.

On the other hand, if m

≡ 1 mod 4 then we can choose a, b to be integers or

half-integers. Thus we can choose b so that

ky − bk ≤

1

4

;

and then we can choose a so that

kx − ak ≤

1

2

.

(Note that a must be an integer or half-integer according as b is an integer or
half-integer; so we can only choose a to within an integer.)

If m < 0 this gives

0

≤ k

α

β

− γk ≤

4 + m

16

,

background image

374

3–13

yielding

|k

α

β

− γk| < 1

if m =

−11, −7 or − 3; while if m > 0 then

m

16

≤ k

α

β

− γk ≤

1

4

,

yielding

|k

α

β

− γk| < 1

if m = 5 or 13.

Thus in all the cases listed we can find γ

∈ Z[ω] such that

|k

α

β

− γk| < 1

Multiplying by β,

|kα − βγk| < |kβk|,

which gives the required result on setting

ρ = α

− βγ,

ie

α = βγ + ρ.

C

Now suppose a

6= 0 is an ideal in Z[ω]. Let α ∈ a (α 6= 0) be an element

minimising

|kαk|. (Such an element certainly exists, since |kαk| is a positive

integer.)

Now suppose β

∈ a. By the lemma we can find γ, ρ ∈ Z[ω] such that

β = αγ + ρ

with

|kρk| < |kαk|.

But

ρ = β

− αγ ∈ a.

Thus by the minimality of

|kαk|,

kαk = 0 =⇒ ρ = 0

=

⇒ β = αγ

=

⇒ β ∈ hαi.

Hence

a

=

hαi.

J

Remarks:

background image

374

3–14

1. We do not claim that these are the only cases in which Q(

m) — or rather

the ring of integers in this field — is a unique factorisation domain. There
are certainly other m for which it is known to hold; and in fact is not known
if the number of such m is finite or infinite. But the result is easily estab-
lished for the m listed above.

2. On the other hand, unique factorisation fails in many quadratic fields. For

example, if m =

−5 then

6 = 2

· 3 = (1 +

−5)(1 −

−5)

Now 2 is irreducible in Z[

5], since

a

2

+ 5b

2

= 2

has no solution in integers. Thus if there were unique factorisation then

2

| 1 +

−5 or 2 | 1 −

−5,

both of which are absurd.

As an example of a real quadratic field in which unique factorisation fails,
consider m = 10. We have

6 = 2

· 3 = (4 +

10)(4

10)

The prime 2 is again irreducible; for

a

2

− 10b

2

=

±2

has no solution in integers, since neither

±2 is a quadratic residue mod

10. (The quadratic residues mod10 are 0,

±1, ±4, 5.) Thus if there were

unique factorisation we would have

2

| 4 +

10

or

2

| 4 −

10,

both of which are absurd.

3.6

The splitting of rational primes

Throughout n this section we shall assume that the integers Z[ω] in Q(

m) form

a principal ideal domain (and so a unique factorisation domain).

Proposition 3.10 Let p

∈ N be a rational prime. Then p either remains a prime

in Z[ω], or else

p =

±π¯π,

where π is a prime in Z[ω]. In other words, p has either one or two prime factors;
and if it has two then these are conjugage.

background image

374

3–15

Proof

I

Suppose

p = π

1

· · · π

r

.

Then

1

k · · · kπ

r

k = kpk = p

2

.

Since

i

k is an integer 6= 1, it follows that either r = 1, ie p remains a prime, or

else r = 2 with

1

k = ±p, kπ

2

k = ±p.

In this case, writing π for π

1

,

p =

±kπk = ±π¯π.

J

We say that p splits in Q(

m) in the latter case, ie if p divides into two prime

factors in Z[ω]. We say that p ramifies if these two prime factors are equal, ie if

p = π

2

,

Corollary 3.2 The rational prime p

∈ N splits if and only if there is an integer

α

∈ Z[ω] with

kαk = ±p.

Proposition 3.11 Suppose p

∈ N is an odd prime with p - m. Then p splits in

Q

(

m) if and only if m is a quadratic residue modp, ie if and only if

x

2

≡ m mod p

for some x

∈ Z.

Proof

I

Suppose

x

2

≡ m mod p.

Then

(x

m)(x +

m) = pq

for some q

∈ Z.

If now p is prime in Z[ω] (where it is assumed, we recall, that there is unique

factorisation). Then

p

| x −

m

or

p

| x +

m,

both of which are absurd, since for example

p

| x −

m =

⇒ x −

m = p(a + b

m)

=

⇒ pb = −1,

where b is (at worst) a half-integer.

J

It remains to consider two cases, p

| m and p = 2.

background image

374

3–16

Proposition 3.12 If the rational prime p

| m then p ramifies in Q(

m).

Proof

I

We have

(

m)

2

= m = pq,

for some q

∈ Z. If p remains prime then

p

|

m =

⇒ kpk | k

m

k

=

⇒ p

2

| −m,

which is impossible, since m is square-free.

Hence

p =

±π¯π,

and

m = πα

for some α

∈ Z[ω]. Note that α cannot contain ¯π as a factor, since this would

imply that

p =

±π¯π |

m,

which as we have seen is impossible.

Taking conjugates

m = ¯

π ¯

α.

Thus

¯

π

|

m.

Since the factorisation of

m is (by assumption) unique,

¯

π

∼ π,

ie p ramifies.

J

Proposition 3.13 The rational prime 2 remains prime in Z[ω] if and only if

m

≡ 5 mod 8.

Moreover, 2 ramifies unless

m

≡ 1 mod 4.

Proof

I

We have dealt with the case where 2

| m, so we may assume that m is

odd.

Suppose first that

m

≡ 3 mod 4.

In this case

(1

m)(1 +

m) = 1

− m = 2q.

background image

374

3–17

If 2 does not split then

2

| 1 −

m

or

2

| 1 +

m,

both of which are absurd.

Thus

2 =

±π¯π,

where

π = a + b

m

(a, b

∈ Z),

say. But then

¯

π = a

− b

m = π + 2b

m.

Since π

| 2 is follows that

π

| ¯π;

and similarly

¯

π

| π.

Thus

¯

π = π,

where is a unit; and so 2 ramifies.

Now suppose

m

≡ 1 mod 4.

Suppose 2 splits, say

a

2

− mb

2

=

±2,

where a, b are integers or half-integers. If a, b

∈ Z then

a

2

− mb

2

≡ 0, ±1 mod 4,

since a

2

, b

2

≡ 0 or 1 mod 4.

Thus a, b must be half-integers, say a = A/2, b = B/2, where A, B are odd

integers. In this case,

A

2

− mB

2

=

±8.

Hence

A

2

− mB

2

≡ 0 mod 8

But

A

2

≡ B

2

≡ 1 mod 8,

and so

A

2

− mB

2

≡ 1 − m mod 8.

Thus the equation is insoluble if

m

≡ 5 mod 8,

ie 2 remains prime in this case.

background image

374

3–18

Finally, if

m

≡ 1 mod 8

then

1

m

2

·

1 +

m

2

=

1

− m

4

= 2q.

If 2 does not split then

2

|

1

m

2

or

2

|

1 +

m

2

,

both of which are absurd.

Suppose

2 =

±π¯π,

where

π =

A + B

m

2

,

with A, B odd; and

¯

π =

A

− B

m

2

= π

− B

m.

Thus

π

| ¯π =⇒ π | B

m

=

⇒ kπk | kB

m

k

=

⇒ ±2 | B

2

m,

which is impossible since B, m are both odd. Hence 2 is unramified in this case.

J

3.7

Quadratic residues

Definition 3.6 Suppose p is an odd rational prime; and suppose a

∈ Z. Then the

Legendre symbol is defined by

a

p

!

=

0

if p

| a

1

if p - a and a is a quadratic residue modp

−1

if a is a quadratic non-residue modp

Proposition 3.14 Suppose p is an odd rational prime; and suppose a, b

∈ Z.

Then

a

p

!

b

p

!

=

ab

p

!

.

background image

374

3–19

Proof

I

The resul is trivial if p

| a or p | b; so we may suppose that p - a, b.

Consider the group-homomorphism

θ : (Z/p)

×

→ (Z/p)

×

: ¯

x

7→ ¯

x

2

.

Since

ker θ =

{±1}

it follows from the First Isomorphism Theorem that

|im θ| =

p

− 1

2

,

and so

(Z/p)

×

/ im θ ∼

= C

2

=

{±1}.

The result follows, since

im θ =

{¯a ∈ (Z/p)

×

:

a

p

!

= 1

}.

J

Proposition 3.15 Suppose p is an odd rational prime; and suppose a

∈ Z. Then

a

(p

−1)/2

a

p

!

mod p.

Proof

I

The resul is trivial if p

| a; so we may suppose that p - a.

By Lagrange’s Theorem (or Fermat’s Little Theorem)

a

p

−1

≡ 1 mod p.

Thus

a

(p

−1)/2

2

≡ 1 mod p;

and so

a

(p

−1)/2

≡ ±1 mod p.

Suppose a is a quadratic residue, say

a

≡ b

2

mod p.

Then

a

p

−1

2

≡ b

p

−1

≡ 1 mod p.

Thus

a

p

!

= 1 =

⇒ a

p

−1

2

≡ 1 mod p.

background image

374

3–20

As we saw in the proof of Proposition 3.14, exactly half, ie

p

−1

2

of the numbers

1, 2, . . . , p

− 1 are quadratic residues. On the other hand, the equation

x

p

−1

2

− 1 = 0

over the field

F

p

= Z/(p) has at most

p

−1

2

roots. It follows that

a

p

!

= 1

⇐⇒ a

p

−1

2

≡ 1 mod p;

and so

a

p

!

≡ a

p

−1

2

mod p;

J

Corollary 3.3 If p

∈ N is an odd rational prime then

−1

p

!

=

1 if p

≡ 1 mod 4,

−1 if p ≡ 3 mod 4.

Proof

I

By the Proposition,

−1

p

!

≡ (−1)

p

−1

2

mod p.

If

p

≡ 1 mod 4,

say

p = 4m + 1,

then

p

− 1

2

= 2m;

while if

p

≡ 3 mod 4,

say

p = 4m + 3,

background image

374

3–21

then

p

− 1

2

= 2m + 1.

J

It is sometimes convenient to take the remainder r

≡ a mod p in the range

p

2

< r <

p

2

.

We may say that a has negative remainder modp if

p

2

< r < 0.

Thus 13 has negative remainder mod7, since

13

≡ −1 mod 7.

Proposition 3.16 Suppose p

∈ N is an odd rational prime; and suppose p - a.

Then

a

p

!

= (

−1)

µ

,

where µ is the number of numbers among

1, 2a, . . . ,

p

− 1

2

a

with negative remainders.

Suppose, for example, p = 11, a = 7. Then

7

≡ −4, 14 ≡ 3, 21 ≡ −1, 28 ≡ −5, 35 ≡ 2 mod 11.

Thus

µ = 3.

Proof

I

Suppose

1

≤ r ≤

p

− 1

2

.

Then just one of the numbers

a, 2a, . . .

p

− 1

2

a

has remainder

±r.

For suppose

ia

≡ r mod p,

ja

≡ −r mod p.

Then

(i + j)a

≡ 0 mod p =⇒ p | i + j

background image

374

3–22

which is impossible since

1

≤ i + j ≤ p − 1.

It follows (by the Pigeon-Hole Principle) that just one of the congruences

ia

≡ ±r mod p (1 ≤ i ≤

p

− 1

2

)

is soluble for each r.

Multiplying together these congruences,

a

· 2a · · ·

p

− 1

2

a

≡ (−1)

µ

1

· 2 · · ·

p

− 1

2

mod p,

ie

a

p

−1

2

1

· 2 · · ·

p

− 1

2

≡ (−1)

µ

1

· 2 · · ·

p

− 1

2

mod p,

and so

a

p

−1

2

≡ (−1)

µ

mod p.

Since

a

p

!

≡ a

p

−1

2

mod p

by Proposition 3.15, we conclude that

a

p

!

≡ (−1)

µ

mod p.

J

Proposition 3.17 If p

∈ N is an odd rational prime then

2

p

!

=

1 if p

≡ ±1 mod 8,

−1 if p ≡ ±3 mod 8.

Proof

I

Consider the numbers

2, 4, . . . , p

− 1.

The number 2i will have negative remainder if

p

2

< 2i < p,

ie

p

4

< i <

p

2

.

background image

374

3–23

Thus the µ in Proposition 3.16 is given by

µ =

p

2

p

4

.

We consider p mod 8. If

p

≡ 1 mod 8,

say

p = 8m + 1,

then

p

2

= 4m,

p

4

= 2m,

and so

µ = 2m.

If

p

≡ 3 mod 8,

say

p = 8m + 3,

then

p

2

= 4m + 1,

p

4

= 2m,

and so

µ = 2m + 1.

If

p

≡ 5 mod 8,

say

p = 8m + 5,

then

p

2

= 4m + 2,

p

4

= 2m + 1,

background image

374

3–24

and so

µ = 2m + 1.

If

p

≡ 7 mod 8,

say

p = 8m + 7,

then

p

2

= 4m + 3,

p

4

= 2m + 1,

and so

µ = 2m + 2.

J

Corollary 3.4 If p

∈ N is an odd rational prime then

−2

p

!

=

1 if p

≡ 1 or 3 mod 8,

−1 if p ≡ 5 or 7 mod 8.

Proof

I

This follows from the Proposition and the Corollary to Proposition 3.15,

since

−2

p

!

=

−1

p

!

2

p

!

,

by Proposition 3.14.

J

Proposition 3.18 If p

∈ N is an odd rational prime then

3

p

!

=

1 if p

≡ ±1 mod 12,

−1 if p ≡ ±5 mod 12.

Proof

I

If

0 < i <

p

2

then

0 < 3i <

3p

2

.

background image

374

3–25

Thus 3i has negative remainder if

p

2

< 3i < p,

ie

p

6

< i <

p

3

.

Thus

µ =

p

3

p

6

.

If

p

≡ 1 mod 6,

say

p = 6m + 1,

then

p

3

= 2m,

p

6

= m,

and so

µ = m.

If

p

≡ 5 mod 6,

say

p = 6m + 5,

then

p

3

= 2m + 1,

p

6

= m,

and so

µ = m + 1.

The result follows.

J

background image

374

3–26

Corollary 3.5 If p

∈ N is an odd rational prime then

−3

p

!

=

1 if p

≡ 1 mod 6,

−1 if p ≡ 5 mod 6.

Proof

I

This follows from the Proposition and the Corollary to Proposition 3.15,

since

−3

p

!

=

−1

p

!

3

p

!

,

by Proposition 3.14.

J

Proposition 3.19 If p

∈ N is an odd rational prime then

5

p

!

=

1 if p

≡ ±1 mod 10,

−1 if p ≡ ±3 mod 10.

Proof

I

If

0 < i <

p

2

then

0 < 5i <

5p

2

.

Thus 5i has negative remainder if

p

2

< 5i < p

or

3p

2

< i < 2p,

ie

p

10

< i <

p

5

or

3p

10

< i <

2p

5

.

Thus

µ =

p

5

p

10

+

2p

5

3p

10

.

If

p

≡ 1 mod 12,

say

p = 10m + 1,

then

p

5

= 2m,

p

10

= m,

2p

5

= 4m,

3p

10

= 3m,

and so

µ = 2m.

The other cases are left to the reader.

J

background image

374

3–27

3.8

Gauss’ Law of Quadratic Reciprocity

Proposition 3.16 provides an algorithm for computing the Legendre symbol, as
illustrated in Propositions 3.17–3.19, perfectly adequate for our purposes. How-
ever, Euler discovered and Gauss proved a remarkable result which makes com-
putation of the symbol childishly simple. This result — The Law of Quadratic
Reciprocity — has been called the most beautiful result in Number Theory, so it
would be a pity not to mention it, even though — as we said — we do not really
need it.

Proposition 3.20 Suppose p, q

∈ N are two distinct odd rational primes. Then

q

p

!

p

q

!

=

−1 if p ≡ q ≡ 3 mod 4,
1 otherwise.

Another way of putting this is to say that

q

p

!

p

q

!

= (

−1)

p

−1

2

q

−1

2

.

Proof

I

Let

S =

{1, 2, . . . ,

p

− 1

2

}, T = {1, 2, . . . ,

q

− 1

2

}.

We shall choose remainders modp from the set

{−

p

2

< i <

p

2

} = −S ∪ {0} ∪ S,

and remainders modq from the set

{−

q

2

< i <

q

2

} = −T ∪ {0} ∪ T.

By Gauss’ Lemma (Proposition 3.16),

q

p

!

= (

−1)

µ

,

p

q

!

= (

−1)

ν

,

where

µ =

k{i ∈ S : qi mod p ∈ −S}k, ν = k{i ∈ T : pi mod q ∈ −T }k.

By ‘qi mod p

∈ −S’ we mean that there exists a j (necessarily unique) such

that

qi

− pj ∈ −S.

But now we observe that, in this last formula,

0 < i <

p

2

=

⇒ 0 < j <

q

2

.

background image

374

3–28

Figure 3.1: p = 11, q = 7

The basic idea of the proof is to associate to each such contribution to µ the

‘point’ (i, j)

∈ S × T . Thus

µ =

k{(i, j) ∈ S × T : −

p

2

< qi

− pj < 0}k;

and similarly

ν =

k{(i, j) ∈ S × T : 0 < qi − pj <

q

2

}k,

where we have reversed the order of the inequality on the right so that both for-
mulae are expressed in terms of (qi

− pj).

Let us write [R] for the number of integer points in the region R

⊂ R

2

. Then

µ = [R

1

], ν = [R

2

],

where

R

1

=

{(x, y) ∈ R : −

p

2

< qx

−py < 0}, R

2

=

{(x, y) ∈ R : 0 < qx−py <

q

2

},

and R denotes the rectangle

R =

{(x, y) : 0 < x <

p

2

, 0 < y <

p

2

}.

The line

qx

− py = 0

is a diagonal of the rectangle R, and R

1

, R

2

are strips above and below the diago-

nal (Fig 3.8).

This leaves two triangular regions in R,

R

3

=

{(x, y) ∈ R : qx − py < −

p

2

}, R

4

=

{(x, y) ∈ R : qx − py >

q

2

}.

We shall show that, surprisingly perhaps, reflection in a central point sends the

integer points in these two regions into each other, so that

[R

3

] = [R

4

].

Since

R = R

1

∪ R

2

∪ R

3

∪ R

4

,

it will follow that

[R

1

] + [R

2

] + [R

3

] + [R

4

] = [R] =

p

− 1

2

q

− 1

2

,

background image

374

3–29

ie

µ + ν + [R

3

] + [R

4

] =

p

− 1

2

q

− 1

2

.

But if now [R

3

] = [R

4

] then it will follow that

µ + ν

p

− 1

2

q

− 1

2

mod 2,

which is exactly what we have to prove.

It remains to define our central reflection. Note that reflection in the centre

(

p
4

,

q
4

) of the rectangle R will not serve, since this does not send integer points into

integer points. For that, we must reflect in a point whose coordinates are integers
or half-integers.

We choose this point by “shrinking” the rectangle R to a rectangle bounded

by integer points, ie the rectangle

R

0

=

{1 ≤ x ≤

p

− 1

2

, 1

≤ y ≤

q

− 1

2

}.

Now we take P to be the centre of this rectangle, ie

P = (

p + 1

4

,

q + 1

4

).

The reflection is then given by

(x, y)

7→ (X, Y ) = (

p + 1

x,

q + 1

y).

It is clear that reflection in P will send the integer points of R into themselves.

But it is not clear that it will send the integer points in R

3

into those in R

4

, and

vice versa. To see that, let us shrink these triangles as we shrank the rectangle. If
x, y

∈ Z then

qx

− py < −

p

2

=

⇒ qx − py ≤ −

p + 1

2

;

and similarly

qx

− py >

q

2

=

⇒ qx − py ≥

q + 1

2

.

Now reflection in P does send the two lines

qx

− py = −

p + 1

2

, qx

− py =

q + 1

2

into each other; for

qX

− pY = q(p + 1 − x) − p(q + 1 − y) = (q − p) − (qx − py),

and so

qx

− py = −

p + 1

2

⇐⇒ qX − pY = (q − p) +

p + 1

2

=

q + 1

2

.

background image

374

3–30

We conclude that

[R

3

] = [R

4

].

Hence

[R] = [R

1

] + [R

2

] + [R

3

] + [R

4

]

≡ µ + ν mod 2,

and so

µ + ν

≡ [R] =

p

− 1

2

q

− 1

2

.

J

Example: Take p = 37, q = 47. Then

37

47

!

=

47

37

!

since 37

≡ 1 mod 4

=

10

37

!

=

2

37

!

5

37

!

=

5

37

!

since 37

≡ −3 mod 8

=

37

5

!

since 5

≡ 1 mod 4

=

2

5

!

=

−(−1) = 1.

Thus 37 is a quadratic residue mod47.

We could have avoided using the result for

2

p

!

:

10

37

!

=

−27

37

!

=

−1

37

!

3

37

!

3

= (

−1)

18

37

3

!

=

1

3

!

= 1.

3.9

Some quadratic fields

We end by applying the results we have established to a small number of quadratic
fields.

background image

374

3–31

3.9.1

The gaussian field Q(i)

Proposition 3.21

1. The integers in Q(i) are the gaussian integers

a + bi

(a, b

∈ Z)

2. The units in Z[i] are the numbers

±1, ±i.

3. The ring of integers Z[i] is a principal ideal domain (and so a unique fac-

torisation domain).

4. The prime 2 ramifies in Z[i]:

2 =

−i(1 + i)

2

.

The odd prime p splits in Z[i] if and only if

p

≡ 1 mod 4,

in which case it splits into two conjugate but inequivalent primes:

p =

±π¯π.

Proof

I

This follows from Propositions 3.4, 3.7, 3.9, 3.11–3.13, and the Corollary

to Proposition 3.15.

J

Factorisation in the gaussian field Q(i) gives interesting information on the

expression of a number as a sum of two squares.

Proposition 3.22 An integer n > 0 is expressible as a sum of two squares,

n = a

2

+ b

2

(a, b

∈ Z)

if and only if each prime p

≡ 3 mod 4 occurs to an even power in n.

Proof

I

Suppose

n = a

2

+ b

2

= (a + bi)(a

− bi).

Let

a + bi = π

e

1

1

· · · π

e

r

r

.

Taking norms,

n =

ka + bik = kπ

1

k

e

1

· · · kπ

r

k

e

r

.

Suppose

p

≡ 3 mod 4.

Then p remains prime in Z[i], by Proposition 3.21.

background image

374

3–32

Suppose

p

e

k a + ib,

ie

p

e

| a + ib but p

e+1

-

a + ib.

Then

p

e

k a − ib,

since

a + ib = p

e

α =

⇒ a − ib = p

e

¯

α,

on taking conjugates. Hence

p

2e

k n = (a + ib)(a − ib),

ie p appears in n with even exponent.

We have shown, incidentally, that if p

≡ 3 mod 4 then

p

2e

k n = a

2

+ b

2

=

⇒ p

e

| a, p

e

| b.

In other words, each expression of n as a sum of two squares

n = a

2

+ b

2

is of the form

n = (p

e

a

0

)

2

+ (p

e

b

0

)

2

,

where

n

p

2e

= a

02

+ b

02

.

We have shown that each prime p

≡ 3 mod 4 must occur with even exponent

in n. Conversely, suppose that this is so.

Each prime p

≡ 1 mod 4 splits in Z[i], by Proposition 3.21, say

p = π

p

π

p

.

Also, 2 ramifies in Z[i]:

2 =

−i(1 + i)

2

.

Now suppose

n = 2

e

2

3

e

3

5

e

5

· · · ,

where e

3

, e

7

, e

1

1, e

1

9, . . . are all even, say

p

≡ 3 mod 4 =⇒ e

p

= 2f

p

.

background image

374

3–33

Let

α = α

2

α

3

α

5

· · · ,

where

α

p

=

(1 + i)

e

2

if p = 2,

π

e

p

p

if p

≡ 1 mod 4,

p

f

p

if p

≡ 3 mod 4.

Then

p

k = p

e

p

in all cases, and so

kαk =

Y

p

p

k =

Y

p

p

e

p

= n.

Thus if

α = a + bi

then

n = a

2

+ b

2

.

J

It’s worth noting that this argument actually gives the number of ways of ex-

pressing n as a sum of two squares, ie the number of solutions of

n = a

2

+ b

2

(a, b

∈ Z).

For the number of solutions is the number of integers α

∈ Z[i] such that

n =

k(kα) = α¯

α.

Observe that when p

≡ 1 mod 3 in the argument above we could equally well

have taken

α

p

= π

r

¯

π

s

for any r, s

≥ 0 with

r + s = e

p

.

There are just

e

p

+ 1

ways of choosing α

p

in this way.

It follows from unique factorisation that the choice of the α

p

for p

≡ 1 mod 4

determines α up to a unit, ie the general solution is

α = (1 + i)

e

2

Y

p

≡1 mod 4

α

p

Y

p

≡3 mod 4

p

f

p

.

Since there are four units,

±1, ±i, we conclude that the number of ways of ex-

pressing n as a sum of two sqares is

4

Y

p

≡1 mod 4

(e

p

+ 1).

background image

374

3–34

Note that in this calculation, each solution

n = a

2

+ b

2

with

0 < a < b

gives rise to 8 solutions:

n = (

±a)

2

+ (

±b)

2

,

n = (

±b)

2

+ (

±a)

2

.

To these must be added solutions with a = 0 or with a = b. The former occurs

only if n = m

2

, giving 4 additional solutions:

n = 0

2

+ (

±m)

2

= (

±m)

2

+ 0

2

;

while the latter occurs only if n = 2m

2

, again giving 4 additional solutions:

n = (

±m)

2

+ (

±m)

2

.

We conclude that the number of solutions with a, b

≥ 0 is

1
2

Q

p

≡1 mod 4

(e

p

+ 1)

if n

6= m

2

, 2m

2

1
2

Q

p

≡1 mod 4

(e

p

+ 1) + 1

if n = m

2

or 2m

2

.

This is of course assuming that

p

≡ 3 mod 4 =⇒ 2 | e

p

,

without which there are no solutions.

In particular, each prime p

≡ 1 mod 4 is uniquely expressible as a sum of two

squares

n = a

2

+ b

2

(0 < a < b),

eg

53 = 2

2

+ 7

2

.

As another example,

108 = 2

2

3

3

cannot be expressed as a sum of two squares, since e

3

= 3 is odd.

3.9.2

The field Q(

3)

Proposition 3.23

1. The integers in Q(

3) are the numbers

a + b

3

(a, b

∈ Z)

background image

374

3–35

2. The units in Z[

3] are the numbers

±η

n

(n

∈ Z),

where

η = 2 +

3.

3. The ring of integers Z[

3] is a principal ideal domain (and so a unique

factorisation domain).

4. The primes 2 and 3 ramify in Z[

3]:

2 = η

−1

(1 +

3)

2

,

3 = (

3)

2

.

The odd prime p

6= 3 splits in Z[

3] if and only if

p

≡ ±1 mod 12,

in which case it splits into two conjugate but inequivalent primes:

p =

±π¯π.

Proof

I

This follows from Propositions 3.4, 3.8, 3.9, 3.11–3.13, and Proposi-

tion 3.18.

J

3.9.3

The field Q(

5)

Proposition 3.24

1. The integers in Q(

5) are the numbers

a + bω

(a, b

∈ Z),

where

ω =

1 +

5

2

.

2. The units in Z[

5] are the numbers

±ω

n

(n

∈ Z).

3. The ring of integers Z[ω] is a principal ideal domain (and so a unique fac-

torisation domain).

4. The prime 5 ramifies in Z[ω]:

5 = (

5)

2

.

The prime p

6= 5 splits in Z[ω] if and only if

p

≡ ±1 mod 10,

in which case it splits into two conjugate but inequivalent primes:

p =

±π¯π.

Proof

I

This follows from Propositions 3.4, 3.8, 3.9, 3.11–3.13, and Proposi-

tion 3.19.

J

background image

Chapter 4

Mersenne and Fermat numbers

4.1

Mersenne numbers

Proposition 4.1 If

n = a

m

− 1 (a, m > 1)

is prime then

1. a = 2;

2. m is prime.

Proof

I

In the first place,

(a

− 1) | (a

m

− 1);

so if a > 2 then n is certainly not prime.

Suppose m = rs, where r, s > 1. Evidently

(x

− 1) | (x

s

− 1)

in Z[x]; explicitly

x

s

− 1 = (x − 1)(x

s

−1

+ x

s

−2

+ x

s

−3

+

· · · + 1).

Subsitituting x = a

r

,

(a

r

− 1) | (a

rs

− 1) = a

m

− 1.

Thus if a

m

− 1 is prime then m has no proper factors, ie m is prime.

J

Definition 4.1 The numbers

M

p

= 2

p

− 1,

where p is prime, are called Mersenne numbers.

4–1

background image

374

4–2

The numbers

M

2

= 3, M

3

= 7, M

5

= 31, M

7

= 127

are all prime. However,

M

11

= 2047 = 23

· 89.

(It should be emphasized that Mersenne never claimed the Mersenne numbers

were all prime. He listed the numbers M

p

for p

≤ 257, indicating which were

prime, in his view. His list contained several errors.)

The following heuristic argument suggests that there are probably an infinity

of Mersenne primes. (Webster’s Dictionary defines ‘heuristic’ as: providing aid
or direction in the solution of a problem but otherwise unjustified or incapable of
justification
.)

By the Prime Number Theorem, the probability that a large number n is prime

is

1

log n

.

In this estimate we are including even numbers. Thus the probability that an odd
number n is prime is

2

log n

.

Thus the probability that M

p

is prime is

2

p log 2

.

So the expected number of Mersenne primes is

2

log 2

X

1

p

n

where p

n

is the nth prime.

But — again by the Prime Number Theorem —

p

n

≈ n log n.

Thus the expected number of Mersenne primes is

2

log 2

X

1

n log n

=

∞,

since

X

1

n log n

diverges, eg by comparison with

Z

X

1

x log x

= log log X + C.

background image

374

4–3

4.1.1

The Lucas-Lehmer test

Mersenne numbers are important because there is a simple test, announced by
Lucas and proved rigorously by Lehmer, for determining whether or not M

p

is

prime. (There are many necessary tests for primality, eg if p is prime then

2

p

≡ 2 mod p.

What is rare is to find a necessary and sufficient test for the primality of numbers
in a given class, and one which is moreover relatively easy to implement.) For this
reason, all recent “record” primes have been Mersenne primes.

We shall give two slightly different versions of the Lucas-Lehmer test. The

first is only valid if p

≡ 3 mod 4, while the second applies to all Mersenne num-

bers. The two tests are very similar, and equally easy to implement. We are giving
the first only because the proof of its validity is rather simpler. So it should be
viewed as an introduction to the second, and true, Lucas-Lehmer test.

Both proofs are based on arithmetic in quadratic fields: the first in Q(

5), and

the second in Q(

3); and both are based on the following result.

Proposition 4.2 Suppose α is an integer in the field Q(

m); and suppose P is

an odd prime with P - m. Then

α

P

α

if

P

m

!

= 1,

¯

α

if

P

m

!

=

−1.

Proof

I

Suppose

α = a + b

m,

where a, b are integers if m

6≡ 1 mod 4, and half-integers if m ≡ 1 mod 4.

In fact these cases do not really differ; for 2 is invertible modP , so we may

consider a as an integer modP if 2a

∈ Z. Thus

α

P

≡ a

P

+

P

1

!

a

P

−1

b

m +

P

2

!

a

P

−2

bm +

· · · + b

P

m

P

−1
2

m mod P.

Now

P

|

P

r

!

if 1

≤ r ≤ P − 1. Hence

α

P

≡ a

P

+ b

P

m

P

−1
2

m mod P

By Fermat’s Little Theorem,

a

P

≡ a mod P, b

P

≡ b mod P.

background image

374

4–4

Also

m

P

−1
2

m

P

!

mod P,

by Proposition 3.15. Thus

α

P

≡ a + b

P

m

!

m mod P,

ie

m

P

!

= 1 =

⇒ α

P

≡ α mod P,

m

P

!

=

−1 =⇒ α

P

≡ ¯

α mod P.

J

Corollary 4.1 For all integers α in Q(

m,

α

P

2

≡ α mod P.

We may regard this as the analogue of Fermat’s Little Theorem

a

P

≡ a mod P

for quadratic fields.

There is another way of establishing this result, which we shall sketch briefly.

It depends on considering the ring

A = Z[ω]/(P ).

formed by the remainders

α mod P

of integers α in Q(

m).

There are P

2

elements in this ring, since each α

∈ Z[ω] is congruent modP

to just one of the numbers

a + b

m

where a, b

∈ Z and

0

≤ a, b < P.

There are no nilpotent elements in the ring A if P - m; for if α = a + b

m

then

P

| α

2

=

⇒ P | 2ab, P | a

2

+ b

2

m

=

⇒ P | a, b.

Thus

α

2

≡ 0 mod P =⇒ α ≡ 0 mod P,

background image

374

4–5

from which it follows that, if n > 0,

α

n

≡ 0 mod P =⇒ α ≡ 0 mod P,

A ring without non-zero nilpotent elements is said to be semi-simple. It is not

hard to show that a finite semi-simple commutative ring is a direct sum of fields.

Now there is just one field (up to isomorphism) containing p

e

elements for

each prime power p

e

, namely the galois field GF(p

e

). It follows that either

1. Z[ω]/(P ) ∼

= GF(P

2

); or

2. Z[ω]/(P ) ∼

= GF(P )

⊕ GF(P ).

The non-zero elements in GF(p

e

) form a multiplicative group GF(p

e

)

×

with

p

e

− 1 elements. It follows from Legendre’s Theorem that

a

6= 0 =⇒ a

p

e

−1

= 1

in GF(p

e

). Hence

a

p

e

= a

for all a

∈ GF(p

e

).

Thus in the first case,

α

P

2

≡ α

for all α

∈ Z[ω]/(P ); while in the second case we even have

α

P

≡ α

for all α

∈ Z[ω]/(P ), since this holds in each of the constituent fields.

In the first case we can go further. The galois field GF(p

e

) is of characteristic

p, ie

pa = a +

· · · a = 0,

for all ainGF(p

e

). Also, the map

a

7→ a

p

is an automorphism of GF(p

e

). (This follows by essentially the same argument

that we used above to show that α

P

≡ α or ¯

α above.)

In particular, the map

α

7→ α

P

mod P

is an automorphism of our field

Z

[ω]/(P ).

On the other hand, the map

α

7→ ¯

α

background image

374

4–6

is also an automorphism of Z[ω]/(P ), since

P

| α =⇒ P | ¯

α.

Moreover, this is the only automorphism of Z[ω]/(P ) apart from the identity map,
since any automorphism must send

m mod P

7→ ±

m mod P.

The automorphism

α

7→ α

P

mod P

is not the identity map, since the equation

x

P

− x = 0

has at mos P solutions in the field Z[ω]/(P ). We conclude that

α

P

≡ ¯

α mod P.

If Z[ω] is a principal ideal domain the second case arises if and only if P splits,

which by Proposition 3.14 occurs when

m

P

!

= 1.

Explicitly, if

P = π

1

π

2

,

then

Z

[ω]/(P ) ∼

= Z[ω]/(π

1

)

⊕ Z[ω]/(π

2

)

= GF(P )

⊕ GF(P ).

Proposition 4.3 Suppose p

≡ 3 mod 4. Let the sequence r

n

be defined by

r

1

= 3,

r

n+1

= r

2

n

− 2.

Then M

p

is prime if and only if

M

p

| r

p

−1

.

Proof

I

We work in the field Q(

5). By Proposition 3.4, the integers in this field

are the numbers

a + bω

(a, b

∈ Z)

where

ω =

1 +

5

2

.

By Proposition 3.9, there is unique factorisation in the ring of integers Z[ω].

background image

374

4–7

Lemma 4.1 If r

n

is the sequence defined in the Proposition then

r

n

= ω

2

n

+ ω

−2

n

for each n

≥ 1.

Proof of Lemma

B

Let us set

s

n

= ω

2

n

+ ω

−2

n

for n

≥ 0. Then

s

2
n

=

ω

2

n

+ ω

−2

n

2

= ω

2

n+1

+ 2 + ω

−2

n+1

= s

n+1

+ 2,

ie

s

n+1

= s

2
n

− 2.

Also

s

0

= ω + ω

−1

= ω

− ¯

ω

=

5,

and so

s

1

= s

2
0

− 2 = 3.

We conclude that

r

n

= s

n

= ω

2

n

+ ω

−2

n

for all n

≥ 1.

C

Let us suppose first that M

p

is prime. Let us write P = M

p

.

Lemma 4.2 We have

5

P

!

=

−1.

Proof of Lemma

B

Since

2

4

≡ 1 mod 5

it follows that

2

p

≡ 2

3

mod 5

≡ 3 mod 5.

background image

374

4–8

Hence

P = 2

p

− 1 ≡ 2 mod 5;

and so, by Proposition 3.19,

5

P

!

=

−1.

C

It follows from this Lemma and Proposition 4.2 that

α

P

≡ ¯

α mod P

for all α

∈ Z[ω]. In particular,

ω

P

≡ ¯

ω mod P.

Hence

ω

P +1

≡ ω ¯

ω mod P

≡ kωk mod P

≡ −1 mod P.

In other words,

ω

2

p

≡ −1 mod P.

Thus

ω

2

p

+ 1

≡ 0 mod P.

Dividing by ω

2

p

−1

,

ω

2

p

−1

+ ω

−2

p

−1

≡ 0 mod P,

ie

r

p

−1

≡ 0 mod P.

Conversely, suppose P is a prime factor of M

p

. Then

M

p

| r

p

−1

=

⇒ r

p

−1

≡ 0 mod P

=

⇒ ω

2

p

−1

+ ω

−2

p

−1

≡ 0 mod P

=

⇒ ω

2

p

+ 1

≡ 0 mod P

=

⇒ ω

2

p

≡ −1 mod P.

But this implies that the order of ω mod P is 2

p+1

. For

ω

2

p+1

= (ω

2

p

)

2

≡ 1 mod P,

so if the order of ω mod P is d then

d

| 2

p+1

=

⇒ d = 2

e

background image

374

4–9

for some e

≤ p + 1; and if e ≤ p then

ω

2

p

≡ 1 mod P.

On the other hand, by the Corollary to Proposition 4.2,

ω

P

2

≡ ω mod P =⇒ ω

P

2

−1

≡ 1 mod P.

Hence

2

p+1

| P

2

− 1 = (P + 1)(P − 1).

Now

gcd(P + 1, P

− 1) = 2.

It follows that

2

p

| P + 1 or 2

p

| P − 1.

The latter is impossible since

2

p

> M

p

≥ P > P − 1;

while

2

p

| P + 1 =⇒ 2

p

≤ P + 1 =⇒ M

p

= 2

p

− 1 ≤ P =⇒ P = M

p

.

J

Now for the ‘true’ Lucas-Lehmer test. As we shall see, the proof is a little

harder, which is why we gave the earlier version.

Proposition 4.4 Let the sequence r

n

be defined by

r

1

= 4,

r

n+1

= r

2

n

− 2.

Then M

p

is prime if and only if

M

p

| r

p

−1

.

Proof

I

We work in the field Q(

3). By Proposition 3.4, the integers in this field

are the numbers

a + b

3

(a, b

∈ Z).

By Proposition 3.9, there is unique factorisation in the ring of integers Z[

3].

We set

η = 1 +

3,

= 2 +

3.

Lemma 4.3 The units in Z[

3] are the numbers

±

n

(n

∈ N).

background image

374

4–10

Proof of Lemma

B

It is sufficient, by Proposition 3.8, to show that is the smallest

unit > 1. And from the proof of that Proposition, we need only consider units of
the form

a + b

3

with a, b

≥ 0.

Thus the only possible units in the range (1, ) are

3 and 1 +

3 = η, neither

of which is in fact a unit, since

k

3

k = −3, kηk = −2,

whereas a unit must have norm

±1, by Proposition 3.6.

C

Lemma 4.4 If r

n

is the sequence defined in the Proposition then

r

n

=

2

n

−1

+

−2

n

−1

for each n

≥ 1.

Proof of Lemma

B

Let us set

s

n

=

2

n

−1

+

−2

n

−1

for n

≥ 1. Then

s

2
n

=

2

n

−1

+

−2

n

−1

2

=

2

n

+ 2 +

−2

n

= s

n+1

+ 2,

ie

s

n+1

= s

2
n

− 2.

Also

s

1

= +

−1

= + ¯

= 4.

We conclude that

r

n

= s

n

=

2

n

−1

+

−2

n

−1

for all n

≥ 1.

C

Suppose first that P = M

p

is prime.

Lemma 4.5 We have

3

P

!

=

−1.

background image

374

4–11

Proof of Lemma

B

We have

M

p

= 2

p

− 1

≡ (−1)

p

− 1 mod 3

≡ −1 − 1 mod 3
≡ 1 mod 3;

while

M

p

≡ −1 mod 4.

By the Chinese Remainder Theorem there is just one remainder mod12 with

these remainders mod3 and mod4; and that is 7

≡ −5 mod 12. For any odd

prime p,

M

p

≡ 7 mod 12

Hence

3

P

!

=

−1.

by Proposition 3.18,

C

It follows from this Lemma and Proposition 4.2 that

α

P

≡ ¯

α mod P

for all α

∈ Z[

3]. In particular,

P

≡ ¯ mod P.

Hence

P +1

≡ ¯ mod P
≡ kk mod P

≡ 1 mod P.

In other words,

2

p

≡ 1 mod P.

It follows that

2

p

−1

≡ ± mod P.

We want to show that in fact

2

p

−1

≡ −1 mod P.

This is where things get a little trickier than in the first version of the Lucas-

Lehmer test. In effect, we need a number with negative norm. To this end we
introduce

η = 1 +

3.

Lemma 4.6

1.

kηk = −2.

background image

374

4–12

2. η

2

= 2.

Proof of Lemma

B

This is a matter of simple verification:

kηk = 1 − 3 = −2,

while

η

2

= (1 +

3)

2

= 4 + 2

3

= 2.

C

By Proposition /refMersenneLemma,

η

P

≡ ¯

η mod P,

and so

η

P +1

≡ η¯

η

− 2 mod P,

ie

η

2

p

≡ −2 mod P.

By the Lemma, this can be written

(2)

2

p

−1

≡ −2 mod P,

ie

2

2

p

−1

2

p

−1

≡ −2 mod P,

But by Proposition 3.14,

2

P

−1
2

= 2

2

p

−1

−1

2

P

!

mod P

≡ 1 mod P,

by Proposition 3.17, since

P = 2

p

− 1 ≡ −1 mod 8.

Thus

2

2

p

−1

≡ 2 mod P

background image

374

4–13

and so

2

2

p

−1

≡ −2 mod P.

Hence

2

p

−1

≡ −1 mod P.

Thus

2

p

−1

+ 1

≡ 0 mod P.

Dividing by

2

p

−2

,

2

p

−2

+

−2

p

−2

≡ 0 mod P,

ie

r

p

−1

≡ 0 mod P.

Conversely, suppose P is a prime factor of M

p

. Then

M

p

| r

p

−1

=

⇒ r

p

−1

≡ 0 mod P

=

2

p

−2

+

−2

p

−2

≡ 0 mod P

=

2

p

−1

+ 1

≡ 0 mod P

=

2

p

−1

≡ −1 mod P.

But (by the argument we used in the proof of the first Lucas-Lehmer test) this

implies that the order of mod P is 2

p

.

On the other hand, by the Corollary to Proposition 4.2,

P

2

≡ mod P =⇒

P

2

−1

≡ 1 mod P.

Hence

2

p

| P

2

− 1 = (P + 1)(P − 1).

Now

gcd(P + 1, P

− 1) = 2.

It follows that

2

p

−1

| P + 1 or 2

p

−1

| P − 1.

In either case,

2

p

−1

≤ P + 1 =⇒ P ≥ 2

p

−1

− 1 =

M

p

− 1

2

=

⇒ P ≥

M

p

3

=

M

p

P

< 3.

Since M

p

is odd, this implies that

P = M

p

,

ie M

p

is prime.

J

background image

374

4–14

4.1.2

Perfect numbers

Mersenne numbers are also of interest because of their intimate connection with
perfect numbers.

Definition 4.2 For n

∈ N, n > 0 we denote the number of divisors of n by d(n),

and the sum of these divisors by σ(n).

Example: Since 12 has divisors 1, 2, 3, 4, 6, 12,

d(12) = 6, σ(12) = 28.

Definition 4.3 The number n

∈ N is said to be perfect if

σ(n) = 2n,

ie if n is the sum of its proper divisors.

Example: The number 6 is perfect, since

6 = 1 + 2 + 3.

Proposition 4.5 If

M

p

= 2

p

− 1

is a Mersenne prime then

2

p

−1

(2

p

− 1)

is perfect.

Conversely, every even perfect number is of this form.

Proof

I

In number theory, a function f (n) defined on

{n ∈ N : n > 0} is said to

be multiplicative if

gcd(m, n) = 1 =

⇒ f(mn) = f(m)f(n).

If the function f (n) is multiplicative, and

n = p

e

1

1

· · · p

e

r

r

then

f (n) = f (p

e

1

1

)

· · · f(p

e

r

r

).

Thus the function f (n) is completely determined by its value f (p

e

) for prime

powers.

background image

374

4–15

Lemma 4.7 The functions d(n) and σ(n) are both multiplicative.

Proof of Lemma

B

Suppose gcd(m, n) = 1; and suppose

d

| mn.

Then d is uniquely expressible in the form

d = d

1

d

2

(d

1

| m, d

2

| n).

In fact

d

1

= gcd(d, m), d

2

= gcd(d, n).

It follows that

d(mn) = d(m)d(n);

and

σ(mn) =

X

d

|mn

d

=

X

d

1

|m

d

1

X

d

2

|n

d

2

= σ(m)σ(n).

C

Now suppose

n = 2

p

−1

M

p

where M

p

is prime. Since M

p

is odd,

gcd(2

p

−1

, M

p

) = 1.

Hence

σ(n) = σ(2

p

−1

)σ(M

p

).

If P is prime then evidently

σ(P ) = 1 + P.

On the other hand,

σ(P

e

) = 1 + P + P

2

+

· · · + P

e

=

P

e+1

− 1

P

− 1

.

In particular,

σ(2

e

) = 2

e+1

− 1.

Thus

σ(2

p

−1

) = 2

p

− 1 = M

p

,

background image

374

4–16

while

σ(M

p

) = M

p

+ 1 = 2

p

.

We conclude that

σ(n) = 2

p

M

p

= 2n.

Conversely, suppose n is an even perfect number. We can write n (uniquely)

in the form

n = 2

e

m

where m is odd. Since 2

e

and m are coprime,

σ(n) = σ(2

e

)σ(m) = (2

e+1

− 1)σ(m).

On the other hand, if n is perfect then

σ(n) = 2n = 2

e+1

m.

Thus

2

e+1

− 1

2

e+1

=

m

σ(m)

.

The numerator and denominator on the left are coprime. Hence

m = d(2

e+1

− 1), σ(m) = d2

e+1

,

for some d

∈ N.

If d > 1 then m has at least the factors 1, d, m. Thus

σ(m)

≥ 1 + d + m = 1 + d2

e+1

,

contradicting the value for σ(m) we derived earlier.

It follows that d = 1. But then

σ(m) = 2

e+1

= m + 1.

Thus the only factors of m are 1 and m, ie

m = 2

e+1

− 1 = M

e+1

is prime. Setting e + 1 = p, we conclude that

n = 2

p

−1

M

p

,

where M

p

is prime.

J

It is an unsolved problem whether or not there are any odd perfect numbers.
The first 4 even perfect numbers are

2

1

M

2

= 6, 2

2

M

3

= 28, 2

4

M

5

= 496, 2

6

M

7

= 8128.

(In fact these are the first 4 perfect numbers, since it is known that any odd perfect
number must have at least 300 digits!)

background image

374

4–17

4.2

Fermat numbers

Proposition 4.6 If

n = a

m

+ 1

(a, m > 1)

is prime then

1. a2 is even;

2. m = 2

e

.

Proof

I

If a is odd then n is even and > 2, and so not prime.

Suppose m has an odd factor, say

m = rs,

where r is odd. Since x

r

+ 1 = 0 when x =

−1, it follows by the Remainder

Theorem that

(x + 1)

| (x

r

+ 1).

Explicitly,

x

r

+ 1 = (x + 1)(x

r

−1

− x

r

−2

+

· · · − x + 1).

Substituting x = y

s

,

(y

s

+ 1)

| (y

m

+ 1)

in Z[x]. Setting y = a,

(a

s

+ 1)

| (a

rs

+ 1) = (a

m

+ 1).

In particular, a

m

+ 1 is not prime.

Thus if a

m

+ 1 is prime then m cannot have any odd factors. In other words,

m = 2

e

.

J

Definition 4.4 The numbers

F

n

= 2

2

n

+ 1

(n = 0, 1, 2, . . . )

are called Fermat numbers.

Fermat hypothesized — he didn’t claim to have a proof — that all the numbers

F

0

, F

1

, F

2

, . . .

are prime. In fact this is true for

F

0

= 3, F

1

= 5, F

2

= 17, F

3

= 257, F

4

= 65537.

background image

374

4–18

However, Euler showed in 1747 that

F

5

= 2

32

+ 1 = 4294967297

is composite. In fact, no Fermat prime beyond F

4

has been found.

The heuristic argument we used above to suggest that the number of Mersenne

primes is probably infinite now suggests that the number of Fermat primes is
probably finite.

For by the Prime Number Theorem, the probability of F

n

being prime is

≈ 2/ log F

n

≈ 2 · 2

−n

.

Thus the expected number of Fermat primes is

2

X

2

−n

= 4 <

∞.

This argument assumes that the Fermat numbers are “independent”, as far as

primality is concerned. It might be argued that our next result shows that this is
not so. However, the Fermat numbers are so sparse that this does not really affect
our heuristic argument.

Proposition 4.7 The Fermat numbers are coprime, ie

gcd(F

m

, F

n

) = 1

if m

6= n.

Proof

I

Suppose

gcd(F

m

, F

n

) > 1.

Then we can find a prime p (which must be odd) such that

p

| F

m

, p

| F

n

.

Now the numbers

{1, 2, . . . , p − 1} form a group (Z/p)

×

under multiplication

modp. Since p

| F

m

,

2

2

m

≡ −1 mod p.

It follows that the order of 2 mod p (ie the order of 2 in (Z/p)

×

) is exactly 2

m+1

.

For certainly

2

2

m+1

= (2

2

m

)

2

≡ 1 mod p;

and so the order of 2 divides 2

m+1

, ie it is 2

e

for some e

≤ m + 1. But if e ≤ m

then

2

2

m

≡ 1 mod p,

whereas we just saw that the left hand side was

≡ −1 mod p. We conclude that

the order must be 2

m+1

.

background image

374

4–19

But by the same token, the order is also 2

n+1

. This is a contradiction, unless

m = n.

J

We can use this result to give a second proof of Euclid’s Theorem that there

are an infinity of primes.

Proof

I

Each Fermat number F

n

has at least one prime divisor, say q

n

. But by the

last Proposition, the primes

q

0

, q

1

, q

2

, . . .

are all distinct.

J

We end with a kind of pale imitation of the Lucas-Lehmer test, but now applied

to Fermat numbers.

Proposition 4.8 The Fermat number

F

n

= 2

2

n

+ 1

is prime if and only if

3

Fn−1

2

≡ −1 mod F

n

.

Proof

I

Suppose P = F

n

is prime.

Lemma 4.8 We have

F

n

≡ 5 mod 12.

Proof of Lemma

B

Evidently

F

n

≡ 1 mod 4;

while

F

n

≡ (−1)

2

n

+ 1 mod 3

≡ 2 mod 3.

By the Chinese Remainder Theorem these two congruences determine F

n

mod

12; and observation shows that

F

n

≡ 5 mod 12.

C

It follows from this Lemma, and Proposition 3.18, that

3

P

!

=

−1.

Hence

3

P

−1
2

≡ −1 mod P

background image

374

4–20

by Proposition 3.14.

Conversely, suppose

3

Fn−1

2

≡ −1 mod F

n

;

and suppose P is a prime factor of F

n

. Then

3

Fn−1

2

≡ −1 mod P,

ie

3

2

2n

−1

≡ −1 mod P.

It follows (as in the proof of the Lucas-Lehmer theorems) that the order of 3 mod
P is

2

2

n

.

But by Fermat’s Little Theorem,

3

P

−1

≡ 1 mod P.

Hence

2

2

n

| P − 1,

ie

F

n

− 1 | P − 1.

Since P

| F

n

this implies that

F

n

= P,

ie F

n

is prime.

J

This test is more-or-less useless, even for quite small n, since it will take an

inordinate time to compute the power, even working modulo F

n

. However, it does

give a short proof — which we leave to the reader — that F

5

is composite.

It may be worth noting why this test is simpler than its Mersenne analogue.

In the case of Mersenne primes P = M

p

we had to introduce quadratic fields

because the analogue of Fermat’s Little Theorem,

α

P

2

−1

≡ 1 mod P,

then allowed us to find elements of order P + 1 = 2

p

. In the case of Fermat primes

P = F

n

Fermat’s Little Theorem

a

P

−1

= a

2

2n

≡ 1 mod P

suffices.

background image

Chapter 5

Primality

5.1

The Fermat test

Suppose p is an odd prime; and suppose gcd(a, p) = 1, ie p - a. Then

a

p

−1

≡ 1 mod p

by Fermat’s Little Theorem.

Definition 5.1 Suppose n is an odd number > 1. Then we say that n is a pseudo-
prime to base a (or an a-pseudoprime) if

a

n

−1

≡ 1 mod n.

Fermat’s Little Theorem can be restated as

Proposition 5.1 If n is an odd prime then it is a pseudoprime to all bases a co-
prime to
n.

This provides a necessary test for primality, which we may call the Fermat

test.

It is reasonable to suppose that if we perform the test repeatedly with coprime

bases then the results will be independent; so each success will increase the prob-
ability that n is prime — while a failure of course will prove that n is composite.

Unfortunately, there is a flaw in this argument. The test may succeed for all

bases coprime to n even if n is composite.

5.2

Carmichael numbers

Definition 5.2 Suppose n is an odd number > 1. Then we say that n is a Carmichael
number if n is not a prime, but is a pseudoprime to all bases a coprime to n, ie

gcd(a, n) = 1 =

⇒ a

n

−1

≡ 1 mod n.

5–1

background image

374

5–2

Recall the definition of Euler’s function φ(n): for n

≥ 1,

φ(n) =

k{1 ≤ i ≤ n : gcd(i, n) = 1}k,

ie φ(n) is the number of congruence classes modn coprime to n:

Thus

φ(1) = 1, φ(2) = 1, φ(3) = 2, φ(4) = 2, φ(5) = 4, φ(6) = 2, . . . .

Euler’s function is multiplicative in the number-theoretic sense:

gcd(m, n) = 1 =

⇒ φ(mn) = φ(m)φ(n).

For according to the Chinese Remainder Theorem, each pair of remainders a mod
m, b mod n determines a unique remainder c mod mn; and it is easy to see that

gcd(c, mn) = 1

⇐⇒ gcd(a, m) = 1 and gcd(b, n) = 1.

If p is a prime then

φ(p

e

) = p

e

−1

(p

− 1).

For i is coprime to p

e

unless p

| i. Thus all the numbers i ∈ [1, p

e

] are coprime to

p

e

except for the p

e

−1

multiples of p. Hence

φ(p

e

) = p

e

− p

e

−1

= p

e

−1

(p

− 1).

Putting together these results, we see that if

n = p

e

1

1

· · · p

e

r

r

then

φ(n) = p

e

1

−1

1

(p

1

− 1) · · · p

e

r

−1

r

(p

r

− 1).

The congruence classes mod n form a ring Z/(n) with n elements ¯

0, ¯

1, . . . , n

− 1.

The invertible elements (or units) in this ring form a multiplicative group

(Z/n)

×

.

The importance of Euler’s function for us is that this group contains φ(n)

elements:

k(Z/n)

×

k = φ(n).

This follows from the fact that ¯

a is invertible modn if and only if gcd(a, n) = 1.

For certainly ¯

a cannot be invertible if gcd(a, n) = d > 1: if

ab

≡ 1 mod n

then

d

| a, d | n =⇒ d | 1.

background image

374

5–3

Conversely, suppose gcd(a, n) = 1. Consider the map

¯

x

7→ ax : Z/(n) → Z/(n).

This map is injective, since

ax = 0 =

⇒ n | ax =⇒ n | x =⇒ ¯

x = 0.

It is therefore surjective; and in particular

¯

x = ax = 1

for some ¯

x, ie ¯

a is invertible.

But now it follows from Lagrange’s Theorem on the order of elements in finite

groups that

a

φ(n)

≡ 1 mod n

for all a coprime to n. (We may regard this as an extension of Fermat’s Little
Theorem to composite moduli.)

Proposition 5.2 The integer n > 1 is a Carmichael number if and only if

1. n is square-free, ie

n = p

1

· · · p

r

where p

1

, . . . , p

r

are distinct primes; and

2. For each i (1

≤ i ≤ r),

p

i

− 1|n − 1.

Proof

I

Suppose first that n has these properties; and suppose that gcd(a, n) = 1.

Then gcd(a, p

i

) = 1 for each i, and so

a

p

i

−1

≡ 1 mod p

i

,

by Fermat’s Little Theorem. Hence

a

n

−1

≡ 1 mod p

i

since p

i

− 1|n − 1.

Since this holds for all i,

a

n

−1

≡ 1 mod n.

Thus n is a Carmichael number.

Suppose conversely that n is a Carmichael number. First we show that n is

square-free.

Lemma 5.1 Suppose A is an abelian group; and suppose p

| kAk, where p is a

prime. Then A contains an element of order p.

background image

374

5–4

Proof of Lemma

B

We argue by induction on

kAk. The result follows by La-

grange’s Theorem if

kAk = p.

If

kAk > p, take any element a ∈ A, a 6= 0. Suppose a is of order e. If p | e,

say

e = pr

then a

r

is of order p.

If p - e, let B be the quotient-group

B = A/

hai.

Since

p

| kBk = kAk/e

it follows from the inductive hypothesis that B has an element, ¯

a say, of order p.

Then the order of a is a multiple of p, say pr, and a

r

has order p, as before.

C

Remark: In fact this result holds for any finite group G: if p

| kGk then G contains

an element of order p. This follows from Sylow’s Theorem.

In the abelian case the result also follows immediately from the Structure The-

orem for Finite Abelian Groups, which states that such a group A is a product of
cyclic groups of prime-power order:

A = Z/(p

e

1

1

)

⊕ · · · ⊕ Z/(p

e

r

r

).

If p

| kAk then p = p

i

for some i; and p

e

−1

is an element of order p in Z/(p

e

).

Returning to the proof of the Proposition, if a prime, say p = p

1

, occurs as a

square or higher power in n, then

p

|φ(n).

Hence, by the Lemma, there is an element a of order p in (Z/n)

×

. Since

a

n

−1

≡ 1 mod n,

it follows that

p

| n − 1,

which cannot be true since p

| n.

Thus

n = p

1

· · · p

r

,

where p

1

, . . . , p

r

are distinct primes.

Recall that the exponent e of a finite group G is the smallest number e > 0

such that

g

e

= 1

for all g

∈ G. By Lagrange’s Theorem,

e

| kGk.

background image

374

5–5

Lemma 5.2 If p is a prime then the exponent of the group (Z/p)

×

is p

− 1.

Proof of Lemma

B

Suppose G = (Z/p)

×

has exponent e. Then the p

− 1 elements

¯

a

∈ G are all roots of the polynomial equation

x

e

− 1 = 0

over the field

F

p

= Z/(p).

But a polynomial equation of degree d has at most d roots. hence

p

− 1 ≤ e.

Since e

|p − 1 it follows that

e = p

− 1.

C

Remark: It is not hard to show that an abelian group of exponent e must contain
an element of order e. It follows that the group (Z/p)

×

is cyclic. (The generators

of this group are called primitive roots modp.) However, the Lemma above is
sufficient for our purposes.

Returning to the proof of the Proposition, suppose a is coprime to p

i

. By the

Chinese Remainder Theorem we can find b such that

b

≡ a mod p

i

,

b

≡ 1 mod p

j

(j

6= i).

Then b is coprime to n. Hence

b

n

−1

≡ 1 mod n,

since n is a Carmichael number. Thus

a

n

−1

≡ b

n

−1

≡ 1 mod p

i

so if e is the exponent of the group (Z/p)

×

then

e

| n − 1.

Hence, by the Lemma,

p

i

− 1 | n − 1.

J

Example: Let

n = 3

· 11 · 17 = 561.

Then

n

− 1 = 560 = 2

4

· 5 · 7.

Since

3

− 1, 11 − 1, 17 − 1 | n − 1 = 560,

n = 561 is a Carmichael number.

It was generally believed that there were only a finite number of Carmichael

numbers, until Pomerance et al proved in 1993 that there are in fact an infinite
number.

background image

374

5–6

5.3

The Miller-Rabin test

Proposition 5.3 Suppose p is an odd prime. Let

p

− 1 = 2

e

m,

where m is odd. Suppose gcd(a, n) = 1. Then either

a

m

≡ 1 mod n

or else

a

2

i

m

≡ −1 mod n

for some i with 0

≤ i ≤ e − 1.

Proof

I

By Fermat’s Little Theorem,

a

p

−1

≡ 1 mod p.

Thus

a

p

−1

2

2

≡ 1 mod p.

Hence

a

p

−1

2

≡ ±1 mod p.

We know how to distinguish these two cases:

a

p

−1

2

a

p

!

mod p,

by Proposition 3.15.

But now suppose

a

p

−1

2

≡ 1 mod p,

which as we have seen is the case if a is a quadratic residue modp; and suppose
p

≡ 1 mod 4. Then

a

p

−1

4

2

≡ 1 mod p;

and so

a

p

−1

4

≡ ±1 mod p.

Repeating this argument, we either reach a point where we cannot divide the

exponent by 2, ie the exponent has been reduced to m and

a

m

≡ 1 mod n;

or else

a

2

i

m

≡ −1 mod n

for some i

∈ [0, e − 1].

J

background image

374

5–7

Definition 5.3 Suppose n is an odd integer > 1. Let

n

− 1 = 2

e

m,

where m is odd. Suppose gcd(a, n) = 1. Then n is said to be a strong pseudoprime
to base a if either

a

m

≡ 1 mod n

or else

a

2

i

m

≡ −1 mod n

for some i with 0

≤ i ≤ e − 1.

We can re-state the last Proposition as

Proposition 5.4 An odd prime p is a strong pseudoprime to each base a with
gcd(a, p) = 1.

Proposition 5.5 Suppose n is an odd integer > 1. If n is a strong pseudoprime to
each base
a with gcd(a, n) = 1 then n is prime.

Proof

I

Suppose n is composite. Then either n is a prime-power,

n = p

e

(e > 1),

or else n has two distinct prime factors, p and q.

Let us deal with the second case first. Suppose gcd(a, n) = 1. Let the orders

of a modulo p, q, n be r, s, t, respectively. Then

r

| t,

s

| t,

since p

| n, q | n.

We are actually interested only in the powers of 2 dividing these orders. Let

us set

v

2

(u) = e

if

2

e

k u,

ie 2

e

is the highest power of 2 dividing u. Then

v

2

(r)

≤ v

2

(t),

v

2

(s)

≤ v

2

(t),

since r

| t, s | t.

Lemma 5.3 Suppose n is a pseudoprime to base a, ie

a

n

−1

≡ 1 mod n.

Then

v

2

(t)

≤ v

2

(n

− 1).

background image

374

5–8

Proof of Lemma

B

We have

a

n

−1

≡ 1 mod n =⇒ t | n − 1

=

⇒ v

2

(t)

≤ v

2

(n

− 1).

C

Lemma 5.4 Suppose p is an odd prime; and suppose gcd(a, p) = 1. Let the order
of
a mod p be r. Then

v

2

(r)

< v

2

(p

− 1) if

p

a

!

= 1,

= v

2

(p

− 1) if

p

a

!

=

−1.

Proof of Lemma

B

By Proposition 3.14,

a

p

−1

2

p

a

!

mod p.

Thus if

p

a

!

= 1

then

r

|

p

− 1

2

=

⇒ v

2

(r)

≤ v

2

p

− 1

2

= v

2

(p

− 1) − 1.

On the other hand if

p

a

!

=

−1

then

a

p

−1

≡ 1 mod p,

a

p

−1

2

6≡ 1 mod p.

Thus

r

| p − 1,

r -

p

− 1

2

.

It follows that

v

2

(r) = v

2

(p

− 1).

C

By the Chinese Remainder Theorem we can find a coprime to n such that

p

a

!

=

−1,

q

a

!

= 1,

ie a is a quadratic residue modq, and a quadratic non-residue modp.

By the last Lemma,

0

≤ v

2

(s) < v

2

(r) = v

2

(p

− 1) ≤ v

2

(t).

background image

374

5–9

Now suppose a is a strong pseudoprime to base n. Let

n

− 1 = 2

e

m,

where m is odd. If

a

m

≡ 1 mod n

then a has odd order modn, ie

v

2

(t) = 0.

Hence a has odd order modp, ie

v

2

(r) = 0.

But that is impossible, since

v

2

(r) = v

2

(p

− 1) > 0.

Thus

a

2

i

m

≡ −1 mod n

for some i

∈ [0, e). Hence

a

2

i

m

≡ −1 mod p,

a

2

i

m

≡ −1 mod q.

Lemma 5.5 Suppose

a

2

i

m

≡ −1 mod n,

where m is odd. Let the order of a mod n be t. Then

v

2

(t) = i + 1.

Proof of Lemma

B

We have

a

2

i+1

m

=

a

2

i

m

2

≡ 1 mod n.

Hence

t

| 2

i+1

m,

t - 2

i

m.

It follows that

v

2

(t) = i + 1.

C

Applying this Lemma with moduli p, q, n,

v

2

(r) = v

2

(s) = v

2

(t) = i + 1.

But that is a contradiction, since

v

2

(s) < v

2

(p

− 1) = v

2

(r).

We conclude that n is not a strong pseudoprime to base a.

J

background image

374

5–10

5.4

The Jacobi symbol

If p is an odd prime and gcd(a, p) = 1 then then

a

p

−1

2

a

p

!

mod p,

by Proposition 3.15.

We cannot use this as a test of primality as it stands, since the Legendre symbol

has only been defined when p is prime. Jacobi’s extension of the Legendre symbol
overcomes this problem.

Definition 5.4 Suppose n

∈ N is odd. Let

n = p

1

· · · p

r

,

where p

1

, . . . , p

r

are primes (not necessarily distinct). Then we set

a

n

!

=

a

p

1

!

· · ·

a

p

r

!

.

Remarks:

1. Note that Jacobi’s symbol does extends the Legendre symbol; if n is prime

the two coincide.

2. Note too that

a

n

!

= 0

if a, n are not coprime.

3. Suppose

n = p

e

1

1

· · · p

e

r

r

.

Then a is a quadratic residue modn if and only if it is a quadratic residue
modp

e

i

i

for i = 1, . . . , r.

This implies that a is a quadratic residue modp

i

for each i; and so

a

n

!

= 1.

But the converse does not hold;

a

n

!

= 1

does not imply that a is a quadratic residue modn.

background image

374

5–11

For example,

8

15

!

=

8

3

!

8

5

!

=

2

3

!

3

5

!

=

−1 · −1 = 1,

while 8 is not a quadratic residue mod15 since it is not a quadratic residue
mod3.

Many of the basic properties of the Legendre symbol carry over to the Jacobi

symbol, as the next few Propositions show.

Proposition 5.6

1. If m, n

∈ N are both odd then

a

mn

!

=

a

m

!

a

n

!

.

2. For all a, b,

ab

n

!

=

a

n

!

b

n

!

.

Proof

I

The first result follows at once from the definition. The second follows

from the corresponding result for the Legendre symbol.

J

Proposition 5.7 If

a

≡ b mod n

then

a

n

!

=

b

n

!

.

Proof

I

This follows from the corresponding result for the Legendre symbol,

since

a

≡ b mod n =⇒ a ≡ b mod p

i

for each p

i

| n.

J

Proposition 5.8 Suppose m, n

∈ N are odd. Then

n

m

!

=

m

n

!

if m

≡ 1 mod 4 or n ≡ 1 mod 4,

-

m

n

!

if m

≡ n ≡ 3 mod 4.

background image

374

5–12

Proof

I

If m, n are not coprime then both sides are 0; so we may assume that

gcd(m, n) = 1. We have to show that

m

n

!

n

m

!

= (

−1)

m

−1

2

·

n

−1

2

.

Suppose

m = p

1

· · · p

r

,

n = q

1

· · · q

s

(where the primes in each case are not necessarily distinct). By Proposition 5.6,

m

n

!

n

m

!

=

Y

i,j

p

i

q

j

!

p

i

q

j

!

=

Y

i,j

(

−1)

pi−1

2

·

qj −1

2

,

by the Quadratic Reciprocity Theorem (Proposition 3.20).

Thus we have to prove that

m

− 1

2

n

− 1

2

X

i,j

p

i

− 1
2

q

j

− 1
2

mod 2,

ie

(m

− 1)(n − 1) ≡

X

i,j

(p

i

− 1)(q

j

− 1) mod 8.

Lemma 5.6 If a, b

∈ Z are odd then

ab

− 1 ≡ (a − 1) + (b − 1) mod 4.

Proof of Lemma

B

Since a, b are odd,

(a

− 1)(b − 1) ≡ mod4,

ie

ab + 1

≡ a + b mod 4,

from which the result follows.

C

It follows by repeated application of the Lemma that

a

1

· · · a

t

− 1 ≡

X

i

(a

i

− 1) mod 4.

In particular,

m

− 1 ≡ (p

1

− 1) + · · · + (p

r

− 1) mod 4.

background image

374

5–13

Since n

− 1 is even, this implies that

(m

− 1)(n − 1) ≡ (p

1

− 1)(n − 1) + · · · + (p

r

− 1)(n − 1) mod 8.

Again, by the Lemma,

n

− 1 ≡ (q

1

− 1) + · · · + (q

s

− 1) mod 4;

and therefore, since p

i

− 1 is even,

(p

i

− 1)(n − 1) ≡ (p

i

− 1)(q

1

− 1) + · · · + (p

i

− 1)(q

s

− 1) mod 8.

Putting these results together,

(m

− 1)(n − 1) ≡

X

i,j

(p

i

− 1)(q

j

− 1) mod 8,

as required.

J

Proposition 5.9 Suppose n

∈ N is odd. Then

−1

n

!

=

1 if n

≡ 1 mod 4,

−1 if n ≡ 3 mod 4.

Proof

I

Suppose

n = p

1

· · · p

r

q

1

· · · q

s

,

where

p

i

≡ 1 mod 4,

q

j

≡ 3 mod 4.

Then

−1

p

i

!

= 1,

−1

q

j

!

=

−1,

and so

−1

n

!

= (

−1)

s

.

On the other hand,

n

≡ 1

r

3

s

mod 4

1 mod 4 if s is even,

3 mod 4 if s is odd.

J

Proposition 5.10 Suppose n

∈ N is odd. Then

2

n

!

=

1 if n

≡ ±1 mod 8,

−1 if n ≡ ±3 mod 8.

background image

374

5–14

Proof

I

Suppose

n = p

1

· · · p

r

q

1

· · · q

s

,

where

p

i

≡ ±1 mod 8,

q

j

≡ ±3 mod 8.

Then

2

p

i

!

= 1,

2

q

j

!

=

−1,

and so

2

n

!

= (

−1)

s

.

On the other hand,

n

≡ (±1)

r

(

±3)

s

mod 8

±1 mod 8 if s is even,
±3 mod 8 if s is odd.

J

5.5

A weaker test

Recall that if p is prime then

a

1
2

(p

−1)

p

a

!

.

We are now in a position to convert this into a test for primality.

Proposition 5.11 Suppose n

∈ N is odd. Then n is prime if and only if

a

1
2

(n

−1)

n

a

!

mod n

for all a coprime to n.

Proof

I

If n is prime then it certainly has the given property.

Suppose conversely that n has this property. We show first that n must be

square-free. For suppose

p

2

| n,

where p is an odd prime.

Let the exponent of (Z/n)

×

be e. Then

p

| φ(n);

and so

p

| e

background image

374

5–15

by Lemma 5.1 to Proposition 5.2. On the other hand,

e

| n − 1

since

a

n

−1

=

a

n

−1

2

2

≡ 1 mod n.

Thus p

| n − 1 and p | n, which is absurd.

Thus n is square-free, say

n = p

1

· · · p

r

,

where p

1

, . . . , p

r

are distinct odd primes.

Our argument runs along the same lines as the proof of Proposition 5.4. Let

n

− 1 = 2

e

m,

p

i

− 1 = 2

e

i

m

i

;

and let us re-arrange the p

i

so that

e

1

= max(e

1

, . . . , e

r

),

ie

v

2

(p

1

− 1) ≥ v

2

(p

i

− 1)

for 1

≤ i ≤ r.

By the Chinese Remainder Theorem, we can find a coprime to n such that

a

p

1

!

=

−1,

a

p

2

!

= 1,

· · ·

a

p

r

!

= 1.

Thus

a

n

!

=

a

p

1

!

· · ·

a

p

r

!

=

−1;

and so

a

n

−1

2

≡ −1 mod n.

Hence

a

n

−1

2

≡ −1 mod p

i

for 1

≤ i ≤ r.

Let the order of a mod n be d; and let the orders of a mod p

i

be d

i

. Then

v

2

(d) = v

2

(d

1

) =

· · · = v

2

(d

r

) = v

2

(n

− 1),

by Lemma 5.5 to Proposition 5.4.

On the other hand,

a

p

1

!

=

−1 =⇒ v

2

(d

1

) = e

1

,

background image

374

5–16

by Lemma 5.4 to Proposition 5.4; while by the same Lemma,

a

p

i

!

= 1 =

⇒ v

2

(d

i

) < e

i

for 2

≤ i ≤ r.

But this is a contradiction, since eg

e

1

≥ e

2

=

⇒ v

2

(d

1

) > v

2

(d

2

).

J

At first sight this seems to offer an additional test for primality, which could

be incorporated into the Miller-Rabin test at the first stage; having determined
whether

a

n

−1

2

≡ ±1 mod n,

we could compute

a

n

!

and see if this gives the same value.

However, the following result shows that this would be a waste of time; the

two values are certain to coincide.

Proposition 5.12 Suppose n is an odd integer > 1. If n is a strong pseudoprime
to base
a then

a

1
2

(n

−1)

=

a

n

!

.

Proof

I

Let

n

− 1 = 2

e

m,

where m is odd.

Suppose first that

a

m

≡ 1 mod n.

Then

a

m

≡ 1 mod n =⇒ a

1
2

(n

−1)

= a

2

e

−1

m

= (a

m

)

2

e

−1

≡ 1 mod n.

On the other hand, a has odd order modn. Hence a has odd order modp for

each prime p

| n. It follows from Lemma 5.4 to Proposition 5.4 that

a

p

!

= 1.

Since that is true for all p

| n,

a

n

!

=

Y

p

a

p

!

= 1.

background image

374

5–17

Now suppose that

a

2

i

m

≡ −1 mod n,

where 0

≤ i ≤ e − 1. Then

a

1
2

(n

−1)

= a

2

e

−1

m

1 if i < e

− 1

−1 if i = e − 1.

Now

a

2

i

m

≡ −1 mod n =⇒ a

2

i

m

≡ −1 mod p

for each p

| n. Let the order of a mod p be r. Then

v

2

(r) = i + 1

by Lemma 5.5 to Proposition 5.4.

Suppose first that i < e

− 1. In that case

v

2

(r) = i + 1 < e = v

2

(p

− 1).

Hence

a

p

!

= 1

by Lemma 5.4 to Proposition 5.4. Since this holds for all p

| n,

a

n

!

= 1.

Thus the result holds in this case.

Finally, suppose i = e

− 1. Then

a

1
2

(n

−1)

= a

2

e

−1

m

= a

2

i

m

≡ −1 mod n.

If

a

p

!

=

−1

then by Lemma 5.4 to Proposition 5.4

v

2

(p

− 1) = i + 1 = e =⇒ p ≡ 1 mod 2

e

, p

6≡ 1 mod 2

e+1

=

⇒ p ≡ 1 + 2

e

mod 2

e+1

.

On the other hand, if

a

p

!

= 1

then by the same Lemma

v

2

(p

− 1) > i + 1 = e =⇒ p ≡ 1 mod 2

e+1

.

background image

374

5–18

Suppose n has r prime factors p with

a

p

!

=

−1.

Then

n

≡ (1 + 2

e

)

r

mod 2

e+1

1 mod 2

e+1

if r is even,

1 + 2

e

mod 2

e+1

if r is odd.

But

2

e

k n − 1,

and so

n

6≡ 1 mod 2

e+1

.

Thus r is odd, and so

a

n

!

= (

−1)

r

=

−1.

So the result holds also in this last case.

J

However, although the weaker test is of no practical value, it does have some

theoretical significance because of the following result.

Proposition 5.13 Suppose n is an odd integer > 1. Then the congruence classes

{¯a ∈ (Z/n)

×

: ¯

a

n

−1

2

=

¯

a

n

!

}

form a subgroup of (Z/n)

×

.

Proof

I

This follows at once from the multiplicative property of the Jacobi sym-

bol, as spelled out in Proposition 5.6(ii).

J

By Proposition 5.11, this subgroup is proper if and only if n is composite. But

it has been shown (by E. Bach) that if the Extended Riemann Hypothesis (ERH)
holds, and

S

⊂ (Z/n)

×

is a proper subgroup then there is an a /

∈ S with

0 < a < 2(log n)

2

.

This implies that if the ERH holds then our weaker test, and so a fortiori the

Miller-Rabin test, must complete in polynomial time; for we need only determine
whether n is a strong a-pseudoprime for a in the above range.


Document Outline


Wyszukiwarka

Podobne podstrony:
Elementary Number Theory Notes D Santos (2004) WW
Elementary Number Theory (Math 780 instructors notes) (1996) WW
Clark Elementary Number Theory
Algebra and Number Theory A Baker (2003) WW
Clinical Advances in Cognitive Psychotherapy Theory and Application R Leahy, E Dowd (Springer, 200
Algebraic Number Theory (Math 784 instructors notes) (1997) WW
Computational Number Theory (math 788 instructors notes) (1986) WW
Analytic Number Theory D Newman (Springer, 1998) WW
Algebra and Number Theory A Baker
Chen Elementary & Analytic Number Theory (1981) [sharethefiles com]
Clinical Advances in Cognitive Psychotherapy Theory and Application R Leahy, E Dowd (Springer, 200
Theory and practise of teaching history 18.10.2011, PWSZ, Theory and practise of teaching history
Introduction relevance theory and literary style
SHSBC 247 R2 12 THEORY AND PRACTICE PART I
aleister crowley magic in theory and practice

więcej podobnych podstron